Prep Test 9

63
KAPLAN LSAT PREP LSAT RELEASED TEST IX EXPLAINED A Guide to the October, 1993 LSAT KAPLAN The answer to the test question.

Transcript of Prep Test 9

Page 1: Prep Test 9

KAPLAN LSAT PREP

LSAT

RELEASED TEST IXEXPLAINED

A Guide to the October, 1993 LSAT

KAPLANThe answer to the test question.

Page 2: Prep Test 9

1993 Stanley H. Kaplan Educational Center Ltd

All rights reserved. No part of this book may be reproduced in any form, by photostat,microfilm, xerography or any other means, or incorporated into any informationretrieval system, electronic or mechanical, without the written permission of Stanley H.Kaplan Educational Center Ltd.

Page 3: Prep Test 9

© K A P L A N 1

SECTION I:

READING COMPREHENSION

Page 4: Prep Test 9

KAPLAN LSAT PREP ________________________________________________________ LSAT Test IX Explained: Section I

2 © K A P L A N

No science at all in this section, which is nice. No especially dense or difficult prose either,which is even nicer. We honestly cannot say that any one passage’s structure or languageseems so difficult or so challenging that you almost certainly should have left it to last.(Neither, incidentally, could the students we spoke to right after they took the 10/93 exam;their report was that all was pretty normal.) We would point out, however, that of all fourpassages, the first one’s structure seems by far the simplest. If, as you preview the passage,you read “Many argue that” and skim down to “By contrast, others claim that,” then allyou need to do next is figure out where the author stands. So Passage 1 is straightforward;this one seems to be worth doing early. (And if you should happen to stumble across anyone of the several places where the author affirms agreement with Paragraph 1, so muchthe better — the passage becomes easier still.)Incidentally, to those who still resist the idea that you can and should try to pick up a fewpoints after reading the first third or so of the passage: Notice that after reading just thefirst paragraph of Passage 2, you know enough to answer Questions 7, 9, and 10. Three outof seven! And you should hit those early on, since their focus is Paragraph 1 exclusively,and reading other parts of the passage might very well blur your understanding of thatparagraph if you delay working on them.Just thought we’d mention it. Remember that your goal is to rack up points, not to readpassages for the sake of reading them. “Getting through the passage” is a useless objective.Attacking the passage and questions is what makes sense. So the sooner your reading paysoff in points, the better.

Page 5: Prep Test 9

KAPLAN LSAT PREP ________________________________________________________ LSAT Test IX Explained: Section I

© K A P L A N 3

Questions 1 - 6

Don’t forget the value of a quick overview of passage structure. The briefest overview ofthis passage reveals the author’s basic structure: The first paragraph begins with “manyargue that,” the second begins with “by contrast, others claim.” The only thing you don’tknow at that point is whether the author ever comes down on one side. Paragraph 3 isn’tmuch help, since it begins with “For example,” which signals a continuation of some train ofthought in the previous paragraph. In fact, when you look at the end of Paragraph 2 tofigure out, “example of what?” you discover that the testwriters have become rather sneaky.Hidden there at the end of Paragraph 2 is the author taking a position on one side of thecontroversy: Technology is in fact good for art. So the basic passage structure is this:Paragraph 1 describing one side of an argument, Paragraph 2 laying out the other side, andthen Paragraph 3 presenting the author’s argument for the position taken in Paragraph 1.

1. (B) With that structure in mind, answering Question 1 is a breeze: Correct choice (B) isa simple short paraphrase of that last sentence of Paragraph 2. Choice (A) is a hugeoverstatement; the author thinks technology is good for art, but not quite that good!Besides, (A) just doesn’t make sense in the real world. You know it’s not true. While youdon’t want to use outside knowledge, LSAT credited answers will never violate basiccommon sense. (C) is a true statement, but this author’s attitude is positive and this choice isnegative. (D) is another huge overstatement; technology might transform the way thepublic responds to certain kinds of art, like performance art; but not all art. (E) is a truestatement but it’s way too objective and neutral to be correct here. When the author takesa definitively positive position, the correct answer to a main idea question must reflectthat.

2. (A) Read unusually long question stems carefully — don’t assume you know whatthey say. Here the key to the question is the word “opponents” — you’re not just inferringthe author’s opinion of something, as you usually are in inference questions; you’reinferring in what way the author agrees with her opponents. According to Paragraph 2, theopponents of techology worry about two things: that technology impedes creativity, andthat it will eliminate live performances “altogether.” The author then disagrees with thissecond point by saying that we are sufficiently committed to art to preserve the liveperformance. She would not address the worry about live performances by saying that“we” will preserve them unless she believed that they were important. In other words, shemust agree with (A), or she would dismiss the opponents’ fears by saying “Liveperformances aren’t important.”(B) comes from the same sentence as (A); the difference is that the author calls it cynical toassume that the public lacks commitment, so she hardly feels that the public commitmentis “questionable.” (C) also comes from the same part of Paragraph 2, but again the key isunderstanding that the author dismisses the fears of her opponents. She says that heropponents act as if technology is an unprecedented challenge to art — and then calls thatattitude “cynical.” (D) is an opinion that this author holds, but her opponents do not. (E)contradicts the spirit of the main idea of the passage — that art is not vulnerable in somedestructive way to technology, but rather is enhanced by it. Worse yet, it’s outside thescope of the passage. To say that the performing arts are “especially” vulnerable suggestssome sort of contrast with something “less” vulnerable, but this passage sticks exclusivelyto technology and the arts.

Page 6: Prep Test 9

KAPLAN LSAT PREP ________________________________________________________ LSAT Test IX Explained: Section I

4 © K A P L A N

3. (D) When questions deal with the same issues and the same parts of the passage,answer them together. Here the focus is once again on the opponents of new tech in art,specifically on their attitudes towards live performance, which you’ve just finishedaddressing in Question 2. Answer this one too while you’re still focused on that part of thepassage and the relevant issues. And bring your Logical Reasoning skills into play: Theposition of the opponents is that new tech may alter or even eliminate live performance; toundermine that position, look for a choice proving that new tech and live performance cancoexist peacefully. That’s (D): Not only do live performances continue, they actuallybenefit from new tech.Knowledge of performing artists does not necessarily get people off their couches and intotheaters, so (A) is wrong. (B) strengthens the opponents’ case: If audiences like watching athome better, that will keep them away from live performances. (C) is completelyirrelevant: This is about the effect of live performances on technology, not vice versa! And(E) strengthens the opponents’ position; if musicians become less spontaneous, tech has infact had an adverse effect on art. Just as in Logical Reasoning, a question asking you toweaken an argument is likely to contain distracters that strengthen it...and vice versa.

4. (E) The Steadicam appears in Paragraph 1, the one that lays out the “tech is good”position. While several of the choices have a positive tone towards the relationship of artand technology, (E) is the only one that harmonizes thoroughly with the author’s basicposition: Tech improves, instead of hurting, traditional art forms.(A) is a true statement, but this question is really asking about how the Steadicam supportsthe author’s argument in Paragraph 1. The argument concerns the interrelation of art andtechnology, not the quality of the tech equipment. (B) picks up on a detail in Paragraph 1— that the Steadicam is “uncomplicated” — but that quality isn’t the reason the authormentions the Steadicam. “Technology in general” is way too broad in scope for any part ofthis passage, so (C) is incorrect. (D) picks up an idea from Paragraph 3 — that the cameratransformed Impressionist art and painting in general — and makes it into a generalprinciple this author allegedly supports by talking about the Steadicam. Wrong! Thisfocus on “types of technology” is inappropriate to the passage anyway.

5. (C) “Pro” new tech positions appear in Paragraph 1. In fact, it’s really only one position— that new tech lets artists vary their “forms of expression” and add “new expressions” —followed by a couple of examples of why that’s so. That position is paraphrased by correctchoice (C); in fact it’s right out of line 8.(A) drags in some Luddites; of course, it isn’t the technology itself that opponents object to,it’s the effects of technology on art. It is true, as (B) says, that archives can be expandedthrough technology — but not because they record longer performances, but becauseperformances too difficult to record with old technology can now be recorded. (Longerperformances, indeed! Has it escaped your attention up to now that more than a fewReading Comp. wrong answers are downright silly? No offense if you chose (B), ofcourse.) Choice (D), like (A), brings in an issue not addressed by the passage, let aloneParagraph 1: speed of the creative process. The pace of the process does not concern thisauthor. (E) plays on Paragraph 1’s description of the Steadicam recording of the workingsof a modern dance company. But that’s an example to support the position technologyallows artists to vary their forms of expression. Proponents of technology do not limitthemselves to speculating about its effects on modern dance, as the reference to “video art”in line 4 shows.

Page 7: Prep Test 9

KAPLAN LSAT PREP ________________________________________________________ LSAT Test IX Explained: Section I

© K A P L A N 5

6. (E) Painting since the 19th century is the subject of the long example in Paragraph 3, thesupport for the author’s “thumbs up” on technology and art. Read long answer choicescarefully; often the wrong ones are only partly true, or repeat enticing words andphrases from the passage. For example, Choice (A) is true right up until the “which”clause, where an extraneous issue, how new methods were applied to subjects other thanportraits, pops in. Choice (B) makes a similar mistake, practically quoting lines 46-50, andthen suggesting that new ideas about light and movement resulted in abstract art.Unfortunately, the author says that these new ideas resulted in efforts to make more realisticart. (C) is more or less accurate but for that one unfortunate word, “exclusively.” Whilethe author does say that photography “preempted” portrait painting, she doesn’t say thatthat traditional portrait painting stopped completely. (Common sense tells us that it didno such thing anyhow.) (D) jumbles two ideas from Paragraph 3, that Impressionist artistsgradually broke away from stylistic limitations and that abstract creations resulted fromthe meeting of technology and art. But the author doesn’t connect these two ideas in thepassage, at least not in the cause-and-effect way this answer choice does. Rather, bothphenomena resulted from the collision of photography and portraiture. That leaves (E) —which is similar to choice (D), but different in one important way. It’s right. Here therereally is a cause-and-effect relationship; this choice is a concise paraphrase of lines 56-60.

Page 8: Prep Test 9

KAPLAN LSAT PREP ________________________________________________________ LSAT Test IX Explained: Section I

6 © K A P L A N

Questions 7 - 13

The second passage in this section outlines the post-World War II conflict between thefederal government’s policy of Native American “readjustment,” outlined in Paragraph 1,and an alternative view held by some Native Americans themselves. When you see that apassage is based on a contrast, don’t endeavor to understand all of its nuances. It’susually enough to work out the broad outlines. Here, we see that basically the governmentcalled for greater assimilation of Native Americans into mainstream society, on thegrounds that reservation life may have limited the Constitutional rights of the people. Incontrast, and at about the same time, Native Americans wanted to strengthen tribal unityand identity instead of assimilating.Paragraph 1 ends by promising that the Wisconsin Oneida tribe’s experience with BIA willact as evidence for the clash, and that promise is fulfilled by the rest of the passage. InParagraph 2 we get the details of the BIA's readjustment offer to the Oneida — essentially,inducements to convert the tribe from collective to private land ownership (including thetaxes that, no surprise, accompany it). We find out in Paragraph 3 why the Oneida rejectedthe deal. Structural signals really help you sort out any kind of list, particularly a list of“reasons why” something happened: You might have wanted to seek out and highlight“first suggestion” (l. 36-37), “equally suspicious of” (l. 45), and “Finally” (l. 49), as a way ofsorting out which reason in Paragraph 3 links up to which offer in Paragraph 2. But ofcourse, as lines 33-35 announce, all three reasons basically boil down to the same thing:The tribe had heard it all before. Paragraph 4 hints at the consequences of that rejection,linking up “the lessons of history” (l. 60) with line 35. As you read through a passage,always try to grasp its big idea and structure. This will make the questions much easierto handle. Don't try to memorize the details. Once you get the big idea and structuredown, it'll be easy enough to relocate details.

7. (C) As Paragraph 1 mentions, a central tenet of readjustment was the belief that NativeAmericans should be assimilated into mainstream America. A program designed toencourage Native Americans to move from reservations (whose residents consist solely ofmembers of the same tribe) to urban areas (where they would be certain to mix withindividuals of different backgrounds) would go a long way toward accomplishingreadjustment's goal of assimilating Native Americans into mainstream society. That makeschoice (C) the scenario most consistent with the policy of readjustment.The scenarios presented in each of the wrong choices would have the opposite effect of thatintended by readjustment. Rather than promote assimilation, each of these developmentswould tend to strengthen Native American institutions and identity, an outcome favoredby Native Americans themselves, not by those who advocated readjustment. A commonwrong-answer choice on the LSAT is the choice that contradicts information in thepassage, so be aware of it.

8. (A) The outcome of the ‘56 meeting is the topic of Paragraph 4, so look there. Always,always, always try to figure out which paragraph a given answer is likely to come from,before you start to attack the answer choices. And the more trouble you have locatingthe likely paragraph, the more certain it is that you just don’t have enough of a handle onthe passage structure! Anyhow, after rejecting readjustment in the wake of the meeting,the Oneida asked instead for federal funds for such things as better education and housing.Put another way, they wanted to improve the quality of life for members of the tribe, as

Page 9: Prep Test 9

KAPLAN LSAT PREP ________________________________________________________ LSAT Test IX Explained: Section I

© K A P L A N 7

correct choice (A) says. Remember, correct answers often put a little spin on the passagetext — a bit of paraphrase.Wrong choices (B) and (E) contradict information in the passage. In contrast to what (B)says, the Oneida were already pursuing land claims before the 1956 meeting (claimsmentioned in Paragraph 3, in fact). And (E) is a readjustment idea: In rejectingreadjustment, the Oneida were attempting to maintain their distinct lifestyle, not to pursueone similar to that of other Americans. (C) is outside the scope of the passage: We're nevertold what position the Oneida took on their political status, before or after the 1956meeting. Answer choices that raise issues that the passage does not grapple withsometimes appear . Don't waste time trying to track down an issue that seems to you asif it came out of nowhere. It probably did. Finally, while we're told that Native Americansin general wanted to develop tribal institutions in order to preserve their traditional wayof life, the passage never specifically says that the Oneida sought new tribal institutionsafter the 1956 meeting, eliminating (E).Never answer a question on a hunch. If you're unsure about what the passage says aboutsomething, go back to the text and find the answer. And again, use your knowledge of thepassage's structure to guide your search.

9. (A) As we've already mentioned, the role of Paragraph 1 is to describe two policiestoward Native Americans that were in conflict during the 1940s and 1950s, setting the stagefor the description of how this conflict played itself out in the specific instance of theWisconsin Oneida vs. the BIA. In more abstract terms, which is what Question 9 wants,Paragraph 1 explains the cause of a specific conflict discussed later in the passage. Thatmakes choice (A) the correct answer.Of the wrong choices, (B) is perhaps the most difficult to eliminate. Paragraph 1 doespresent two positions, those of the federal government and Native Americans onreadjustment. But so does the rest of the passage, which presents both sides of the story inthe description of the BIA-Oneida negotiations over readjustment. True, the author sideswith the Oneida, but it's an exaggeration for (B) to say that he “defends” their case to theexclusion of that made by the BIA. Moreover, this choice is inferior to choice (A) becauseit doesn't allude to the 1956 meeting that is the topic of the remaining paragraphs. Far fromcomparing interpretations of a historical conflict (C), Paragraph 1 provides backgroundinformation necessary to comprehend a historical conflict. By the same token, Paragraph 1explains why a historical event unfolded the way it did; it includes no “analysis” of causesand certainly includes no future predictions (D). Finally, (E) is completely off base.Paragraph 1 a history of the BIA? The agency isn’t even mentioned until line 18.

10. (C) Pick up the phrase “increased awareness” in line 6, recognize through the“because” that the phrase is providing evidence for a conclusion, read a few more lines,and you’re home free. This awareness led to a feeling that reservation life might limit theConstitutional rights of Native Americans and, in turn, to the “readjustment movement” (l.8-9). (C) sums that up pithily.The author does contrast the readjustment movement with another social phenomenon (A),namely the Native American effort to assert unity and identity, but doesn’t begin to do sountil line 13, which is too far removed from the civil rights reference. Watch out forchoices that are true (according to the passage), but irrelevant to the question at hand.Regarding (B), increased awareness of civil rights played a part in determininggovernment policy, not that of Native Americans. In fact, we're never told why NativeAmericans were so concerned to protect their unity and identity, just that they were

Page 10: Prep Test 9

KAPLAN LSAT PREP ________________________________________________________ LSAT Test IX Explained: Section I

8 © K A P L A N

concerned with these things. The text doesn't tell us anything about the specific motives ofBIA bureaucrats (D); we know only that they supported the policy of readjustment. InReading Comp., as in Logical Reasoning, choices that question people’s motives tend tobe incorrect. As for (E), if anything the author sides with Native Americans againstreadjustment, and certainly has no advocative purpose in Paragraph 1 or anywhere else.Why the heck would he be fostering support for a policy dating back more than a quarterof a century, anyway?

11. (B) We already know that readjustment advocates believed that the federalgovernment should end its special involvement in Native American affairs. (B) makes thispoint, though in a slightly different way. Saying that Native Americans should be treatedby the government in precisely the same way as others are treated is more or lessequivalent to saying that the government should butt out. Again, the correct answer toinference questions often restates in different language an idea contained in the passage.You are rarely asked to make a true inference by taking the passage's logic a step further.Advocates of readjustment would disagree with all four wrong choices for the simplereason that they all endorse the notion that it’s the government’s role to help maintainNative Americans as a community apart or, at least, to provide them with significantsupport . Indeed, it is opponents of readjustment who would be likely to agree with thesentiments expressed in (A), (C), (D), and (E). After all, according to the passage,opponents of readjustment wanted the federal government to improve the quality of lifefor Native Americans while maintaining the special status of Native American land.

12. (A) The treaty in question is mentioned only once, in the last sentence of Paragraph 2,but it is implicitly alluded to at the end of Paragraph 3 and it’s there that the answer toQuestion 12 is to be found. How come? Because (as you’ll recall), each of the offers madeby the BIA in Paragraph 2 got an Oneida reaction, and each reaction is described in turn inParagraph 3. This is an excellent example of how some questions require you to relate twoor more references from different parts of the passage, and you’ve got to stay heads-up oryou’ll blow it. The “lump-sum payment” (l. 29 and again at l. 50) was of course “in lieu of”(l. 29-30) the Canandaigua Treaty provisions, which the Oneida were clearly loath tochange because doing so might put at risk certain ongoing land claims. (A) is a bit vaguebut on target. At least it gets the positive tone right.Of the wrong choices, (B) and (E) can be eliminated on the basis of their negative tone. It’snot the Treaty that was a source of problems (B) but the readjustment policy, and (E) is agenuine au contraire choice — one which gives us the exact opposite of what the questionis asking for. “An obsolete agreement”? Au contraire. (C) and (D) are positive statementsbut poor choices. Since the tribe and the government were already bound together by theCanandaigua Treaty, the delegates would have no reason to view the treaty as a model fora potential future agreement with the federal government (C). And as for (D), it’s reallyreaching to describe the Oneidas’ desire to affirm tribal identity as an aspiration to become“an independent...nation”; and even if that were a fair characterization, the passage neverlinks it up with the Canandaigua Treaty.

Page 11: Prep Test 9

KAPLAN LSAT PREP ________________________________________________________ LSAT Test IX Explained: Section I

© K A P L A N 9

13. (B) Treat a Reading Comp. parallel reasoning question like any other; there should beno difference in your approach. We’re fortunate in that this one picks up on the issues wewere just exploring in Question 12. Lines 29-32 describe the payment and lines 49-53describe the refusal. Remember, the rejection was not based on financial grounds, but onthe fact that the $60,000 would alter the terms of the Canandaigua Treaty and jeopardizecertain pending land claims. So a parallel situation would involve someone turning downan offer not on financial grounds, but because it would have dire legal consequences.That’s (B), of course — and notice that the gesture of the Oneidas and that of the employeeshare a certain selflessness that makes the parallel even tighter.None of the other choices reflects this logic. In choices (A), (C) and (D), the student, theteenager, and the customer, respectively, refuse the deals offered to them by the university,the parents, and the car dealer, respectively, purely on financial grounds. In none of thesescenarios is a deal refused for a legal reason (or, for that matter, a selfless one). Finally,although the scenario in choice (E) involves a legal issue, the tenant really refuses thelandlord’s offer because it would inconvenience her, not because accepting it mightjeopardize a legal position.

Page 12: Prep Test 9

KAPLAN LSAT PREP ________________________________________________________ LSAT Test IX Explained: Section I

10 © K A P L A N

Questions 14 - 20

Fortunately for those rusty classical scholars among us, the basic topic of the passage — thedevelopment of literacy in ancient Greek society — is spelled out right there in the firstsentence. Like many LSAT reading passages, this one is structured as an argumentagainst an established theory or point of view. The author’s big idea is that scholars whostudy the development of literacy today have misunderstood the development of literacyin ancient Greece and its effect on the political culture of the day. Note how examiningmerely the first sentence of Paragraph 2 lays out the argument structure, as it were, andtells you clearly where the author stands: Examining Athens “exemplifies” (that means “isevidence for”) how“this sort of confusion” (a reference to whatever confusion was justdescribed) works against really understanding the politics of ancient times. Most readingpassages are in fact extended arguments, and your Logical Reasoning skills can andshould be brought to bear on them.In terms of the building blocks, Paragraph 1 describes the problem: Scholars, particularlysome character named Goody, assume that widespread literacy democratized ancientGreek society by “enabling” (Goody’s word) everyone to access (i.e. read) importantdocuments. But our author’s point of view on this becomes clear at the end of Paragraph 2: Developments such as a written code of law didn’t necessarily bring political change,because the power of interpretation remained with the magistrates. Paragraphs 3 and 4simply reinforce this idea of vested interest. Essentially, by controlling the books, the elitecontinued to maintain legal and political power in ancient Greece.The only other major point of interest is lines 43-49, in which the author may have startledyou by suddenly alluding to modern life in the midst of the discussion of ancient Athens.But given lines 1-11, we shouldn’t have been surprised: The whole reason wemisunderstand the effect of literacy on the ancient world is that we see how literacy affectsour world and project that vision onto the past. Remember what we said regardingQuestion 12, above: You need to be alert to references late in a passage that relatestrongly to earlier parts of the passage.

14. (B) Question 14 is a global question, so we're looking for the choice that capturesboth the content and point of view of the entire passage. The gist of the author’sargument is probably best summarized at the end of Paragraph 4, but really it's implicitthroughout: Contrary to Goody’s claims, literacy did not bring about radical politicalchange, because the elite retained control of the important documents. Choice (B) takesthat idea and summarizes it in a more general and abstract way. True, (B) doesn’t mentionAthens, but then none of the choices does, they’re all written in abstract language. Andafter all, by the author’s own admission Athens is just there to “exemplify” the generalpoint.How democracy grows (A) is not the author’s topic; if anything, it’s how democracy doesnot grow (i.e., it doesn’t necessarily grow from literacy, which is (B)’s point). And (C) takesa brief Paragraph 4 reference to religion in another era, and blows it up into a “main idea.”Pshaw. The phrase “best educated” should have set off warning bells in choice (D) — theauthor focuses on the elite’s control of authoritative documents and books withoutsuggesting that their educational background was an issue; and in any event, any choicewhich ignores the topic of the effect of literacy on democracy should have been rejectedimmediately. (E) does sort of allude to the topic, but distorts the situation further. The

Page 13: Prep Test 9

KAPLAN LSAT PREP ________________________________________________________ LSAT Test IX Explained: Section I

© K A P L A N 11

elite retained power by controlling the written word, not by keeping the populationilliterate.

15. (D) As an inference question, this one is pretty broad, and in fact it’s more global thanspecific: It relates once again to the author’s overall point of view on the development ofliteracy. You should keep the “big idea” with you during all questions, so this one needn’thave been a trial. As we just finished saying, the gist of the author’s argument is thatrudimentary reading ability did not shake up the aristocracy in ancient Greece, because theelite had exclusive access to the most important texts. Choice (D) is a central assumptionhere — that widespread literacy does not mean free access to information. If you needmore convincing, note that lines 5-9 specifically state that the author sees confusionbetween those two things.Of the wrong choices, (A) contradicts the passage pretty directly. The author’s point is thatliteracy doesn’t necessarily bring about any political advances at all. Choice (B)’s tooextreme — it’s never suggested that the Greeks showed no benefits from literacy. Thephrase “rapidly evolving” underscores the problem with (C) — there’s no discussion hereof the rate at which social literacy develops (this is probably a distortion of “at thethreshold of widespread literacy,” lines 1-2). Choice (E), finally, suggests that such peopleabstain from political decisions. This one might be very tempting, because at the end ofParagraph 4 it’s clear that when only the elite possess the books, everybody else “is subjectto the elite’s interpretation” of law. But that concept doesn’t mean that the people don’tvote, or attend town meetings, or respond to polls, any of which could reasonably qualifyas “participat[ing] in political decision making.”

16. (C) Question 16 asks about the reference to the “truly knowledgeable minority”discussed in Paragraph 4. Remember, when dealing with a question about the role of adetail, it's important to understand the context in which that detail appears. So insteadof focusing on the micro issue of the detail itself, start with the macro issue of: Where isthe author at that point in his argument? Well, he has just gotten through explaining thatancient Greek juries had no access to books or documents — a major piece of evidence inhis argument that myth mustn’t seduce us into thinking that because the ancients couldread, they read political literature. “Granted,” line 43, is a signal that the author is about totip his hat to a possible explanation or exception, as in “Your dog is filthy. Granted, thosechildren just threw mud at him, but he’s still a mess.” After reading “Granted, peopletoday” are such-and-such, we wait for the other shoe to drop, and here it drops at line 45:“Yet ...” — the point being that what’s true about our world isn’t necessarily true about theancient one. And looking at the lines 43-51 as a whole, we see that both societies rely on theinterpretations of powerful, knowledgeable elites; but ancient Greek society wasn’t able toget “sociopolitical” information from the kind of popular literature and media that wehave access to today. (C) spells out the underlying point: Our access to media makes usless dependent on experts.Choices (A) exaggerates the power of the media. The author doesn’t suggest that it “closesthe gap” between the majority and the elite; it only makes today’s majority a bit less relianton the elite than in ancient Greece. (We still “rely heavily on” it, line 43.) The phrase“specious information” helps target (B) as a wrong answer — the author never criticizestoday’s media. If anything, he’s glad it’s around. Because the author puts quotes around“popular literature” in line 47 you might have thought that he was being snotty, but that’snot so. Elite membership for the majority (D) may sound like a beautiful idea, but it’s

Page 14: Prep Test 9

KAPLAN LSAT PREP ________________________________________________________ LSAT Test IX Explained: Section I

12 © K A P L A N

outside the scope of the passage. (E) is a distortion, and a pretty implausible one—there’sno connection made here between popular literature and educational achievement.

17. (A) The topic of this question is ancient Greek juries, so it’s back to Paragraph 3 we go.For “all...EXCEPT questions,” two different strategies have proven useful: You can scanthe choices before going back to the passage, to see if any choices jump out as untruestatements; or you can simply and casually re-read the portion of the passage inquestion, and then toss out the choices that you know are true. Practice both approachesto see which you’re more comfortable with. Anyhow, the overall point of Paragraph 3 isthat Greek juries had very little real power because they had no direct access to the codesof law, and most of the “true statement” choices can be eliminated because they’reconsistent with this. We’re told that because they were not permitted access to law books(D), they only heard statutes that were read out to them (B), they had to determine the factsand interpret what they heard (C), and deliver their verdicts without the benefit of groupdiscussion (E). That leaves (A); but in terms of its logic, (A) may have jumped out at youas an obvious distortion. The author argues that juries were somewhat democratic becausethey represented a broad spectrum of the population, not just the lowest social classes.

18. (D) We can make quick work of this question, another that concerns a detail relatingdirectly to the overall argument . Lines 16-22 suggest that the tradition of a “law-giver”stems from the (mistaken) idea that the development of a written law brought about abreak in the aristocratic control of society. Here, as elsewhere, the author disagrees thatliteracy brought about real political change, using the word “mythologizing” to suggestthat the development of a written law did not disrupt the status quo (D). Again, whendirected by a question to a detail, don’t just focus on the detail. That detail is onlyplaced there as part of a context, so think about the context! That’s largely what theLSAT is testing, anyhow — your ability to understand the overall context.Just as the correct answer is firmly in synch with the overall argument, two of the wrongchoices fall several miles outside of the scope of the passage. Greek mythology as a topicis not discussed anywhere else in the passage, and so it’s highly unlikely that the authorwould make a point of it here; if you chose (A) or (E), or even were tempted by them, youwere reading a bit lazily and taking the word “myth” far too literally. No offense. (B)should prompt a vigorous Au contraire! from you: The author mentions the law-giver inorder to dispute its historical importance, overestimated by people like Goody. Finally, thequote at line 21 does suggest the Athenians’ respect for their legal tradition (C); however,until the other shoe drops with the structural signal “But,” line 22, the author hasn’t yetreached the point of the paragraph.

19. (B) Just as with the previous question, you must ask yourself: What’s going on at thegiven point of the passage? We’re directed to the part where the author compares anancient law code to the Latin Bible — “a venerated document but a closed book.” Theauthor’s point once again relates to the inaccessibility of important documents, theimplication being that while ancient law codes were tremendously influential documents,very few people were permitted to read them, contrary to the misinterpretation of Goodyand others like him. (B), in fact, wouldn’t be a bad answer to the question “Which of thefollowing would the author agree with?,” let alone this one.Choice (A) could have been ruled out on logical grounds. Nowhere does the authorsuggest that the fewer people read an ancient document, the more important it wasconsidered. (C) is out on tone and content: The author’s not making a critique of

Page 15: Prep Test 9

KAPLAN LSAT PREP ________________________________________________________ LSAT Test IX Explained: Section I

© K A P L A N 13

superstition in “nondemocratic” societies, they aren’t even part of his scope of interest.(D) is wrong because the influence of ancient laws is not the issue, it’s how accessible theywere. And (E) stretches the analogy too far; it was the elite’s control of texts, and not alanguage barrier, that prevented the majority from reading ancient law codes.

20. (B) This global question, even more than Question 14, demands that we translate theauthor’s specific approach into abstract language. Because such language is a step or tworemoved from the explicit content, it poses a special challenge for many students. Payspecial attention to passage structure and, when appropriate, tone, in trying to translatethe abstract choices back into an approximation of the passage you’ve just read.Reviewing the passage from the top, it should be clear that the author outlines anestablished theory in Paragraphs 1 and 2, and then sets out to disprove it in Paragraphs 3and 4. In other words, he is “point[ing] out the weaknesses” in Goody’s approach, which ischoice (B).Choice (A), of course, is misfocused on the topic of contemporary societies, which onlycome up in the passage because people like Goody are misled by modern life when theytry to analyze the ancient world — which is the real topic. Less so than (A), (C) still givestoo great a weight to the modern world. Again, correcting misconceptions about ancientGreece is the focus of the passage as a whole; today’s world simply contributes to thesource of those misconceptions. The need for a new method of approach (D) is neversuggested; the author seems to consider his own assessment of the evidence satisfactoryenough. And finally, both tone and content are wrong in choice (E): The passage conveysno sense of methodical difficulties here, and no concern about the attitudes of the ancientGreeks either.

Page 16: Prep Test 9

KAPLAN LSAT PREP ________________________________________________________ LSAT Test IX Explained: Section I

14 © K A P L A N

Questions 21 - 27

The final passage in this section compares the political attitudes and institutions ofEngland’s American colonies with those of England itself in the two centuries prior to ourindependence. More than any of the others, this passage really hinges on comparisonand contrast. And as we were saying in the intro to the first passage, don’t try tounderstand the two points of view in depth. Start by understanding them in a broadsense. You can always go back and analyze in more detail as the questions demand, butwhy do more work than you need to?Notice that the passage gets underway by defining the two groups in question (the Englishin America and the ones at home) as “sharing a common political vocabulary,” and almostimmediately announces that one group “had given the words a significance quitedifferent.” When Kaplan says — as we so often do — that you are to read the first thirdof the passage with the greatest intensity, this is what we mean: that you are to look for,and pounce on, just this sort of signal of what the author will be interested in doing in therest of the passage. Thereafter, “reading” the passage becomes more of a matter ofconfirming that your initial impressions were indeed accurate. What a time saver that is!The gist of the rest is that the colonial government was set up much the same way as that ofthe Mother Country (Paragraph 2), but that attitudes towards that setup differed on eitherside of the Atlantic; Paragraph 3 explains how the English in England felt and the lastparagraph sums up the colonials’ views.To tie this all up, basically you should have come away from the passage with the idea thatthe colonists and the English gave different meanings to the same political terms (such as“constitution”) and, as a consequence, perceived similar political institutions verydifferently. That comparison/contrast is the big idea here...

21. (C) ...and no sooner said than done, we are rewarded with a point for choice (C) in thismain idea question. This question is probably one that you could have answered correctlyafter reading Paragraph 1 and a cursory skim of the rest, but even if you waited till late toanswer it, the choice of (C) should be pretty much Q.E.D. Many answers to main ideaquestions go wrong because they’re too broad or too narrow in scope — particularly thelatter. Some wrong choices will refer to only a small segment of the passage, while otherswon’t seem to refer to the specific content of the passage at all. Choice (B) , for instance,sums up Paragraph 2 only, while Colonial attitudes, the subject of (D), come up only inParagraph 4 (not to mention the fact that “increasing 18th century hostility” is somethingwe carry with us from our outside knowledge of the American Revolution, not from thepassage itself). (A) and (E), meanwhile, contradict or at least distort the text. Contrary to(A), the colonists and English did too share a common political vocabulary, though theydiffered over the meanings of the political terms they shared. And contrary to (E), asParagraph 4 suggests, colonial attitudes toward constitutions were influenced by thecontents of the charters granted to the colonies by England, not by English legaldevelopments in the 17th century.

22. (D) Since you’re looking for the statement that is not supported by the passage, thebest way to approach this question is by process of elimination. Eliminate thestatements that you know are supported by the passage, and see what’s left. As always,of course, first figure out which paragraph(s) specifically are likely to contribute to thesearch. Here it’s Paragraph 2, even though the topic of the question is colonial politics

Page 17: Prep Test 9

KAPLAN LSAT PREP ________________________________________________________ LSAT Test IX Explained: Section I

© K A P L A N 15

only, while the paragraph is about what the colonies and England had in common.Anyway, it says there that, at the time, people who didn’t own property couldn’t vote (A),each of the colonies had a representative assembly that resembled Britain’s Parliament (B),and some colonies had Royal Governors (C) who were regarded as similar in function to aking (E). That leaves choice (D), and indeed, the passage never states that colonialassemblies could remove Royal Governors from office. What the passage does say, inParagraph 4, is that sometimes the colonists struggled with these governors in order toprotect their rights, but that’s entirely different from saying that the colonists could removethem from office.Incidentally, we’d like to point out that choice (A) — a wrong answer in that it’s inferablefrom the passage — is in fact the contrapositive of one of the author’s statements, namelyline 24. “Only property holders could vote” can be translated to “If one was a voter, thenone had to be a property holder,” and after reversing and negating the terms, you’re leftwith choice (A). We mention this not because we think it was so difficult to reject (A),but because we wanted to reinforce the idea that translating “only if” statements tosimple “if/then”s, and using the contrapositive, are skills that you should be ready toexercise throughout the exam. You never know when those will be precisely what they’retesting for.

23. (C) When you want to know about English kings — when you want to know aboutEngland period — Paragraph 3 is where you search. The kings’ power relative to that ofParliament, we are told there, was declining from the early 17th century onward. In fact,by the 18th century, Parliament was considered all-powerful. In other words, the kings’power relative to that of Parliament was much greater before the early 17th century thanlater on...which makes (C) correct.Wrong choice (A) may have been a bit tricky, since sentences 3 and 4 of Paragraph 3 implythat English kings pre-1600 were absolute authorities who were not subject to the law.However, that’s very different from saying that they were the source of the law back then.Just because they weren’t subject to the law doesn’t mean that they made it. While choice(B) contradicts information in the passage (if pre-1600 English kings weren’t subject to thelaw then, by definition, they could not have flouted it), you may have been tempted by itanyway, since some tend to associate contempt for law with absolute, contemptuousrulers. Choice (D) is outside the scope of the passage: We’re told only about Parliament’srole in English legal reform, not the role played by the kings, if any. Likewise, we aren’ttold whether English kings, before the early 17th century, had to struggle against peoplewho thought that Parliament should be the absolute power in the land (E), althoughstarting around 1600, obviously, English kings were forced to battle such people. And lost.

24. (C) When looking for evidence, listen for signals of same. Those include “because,”and “since,” and phrases like “The most unmistakable sign of this tendency,” lines 30-31.Of course, the “sign” mentioned there (the announcement in law that the King was subjectto the law) is not what the testmakers chose for the right answer, though they could havedone so. Rather, we are told that “together with” that evidence went another piece — thatby the 18th century, the English had accepted the notion of Parliament as an all-powerfulinstitution. And as an example of that unlimited power we’re given correct choice (C). Agood rule of thumb is not to fall for the first thing in the paragraph that seems like itought to be the answer, but to read a little further on.Since 18th-century Englishmen accepted the absolute power of Parliament, they could not,as (A) alleges, have been uncomfortable with institutions that had absolute power. So (A)

Page 18: Prep Test 9

KAPLAN LSAT PREP ________________________________________________________ LSAT Test IX Explained: Section I

16 © K A P L A N

is a contradiction. Regarding (B), the English may very well have thought that theirinterests would be better taken care of by Parliament than by the King — why else wouldthey accept Parliament’s new power? But you must always remember the question that’sbeing posed. A statement that is consistent with the author’s view — that is, a “true”statement — is not enough. Why the English made Parliament omnipotent goesunmentioned in the passage, so it’s clearly not the evidence used by the author to supportanything. Much the same can be said about (D) and (E). (D) certainly gives a reason forpopular dissatisfaction with a King — although I’m sure you’re aware that royal stupidityhas historically not been a disqualification for leadership, and in any case the passage isdiscussing Kings in general, not one dumb monarch — but in any case it’s simply not citedby the author. Likewise, that the English didn’t object to the expanding power ofParliament may be taken as an indication that they supported governmental reform (E),but it’s not explicitly cited here, and so neither (D) nor (E) can possibly be dubbed“evidence used by the author.”

25. (E) This is one of the tougher questions in the set, because its topic is what the colonistsprobably viewed as a source of their quarrel with England, while the passage is mostlydevoted to the attitudes of colonists and the English about political institutions, and is onlyin passing interested in their views of each other. Also, it’s not immediately apparentwhere in the passage the answer to Question 25 is to be found. For some questions, there’sreally no faster and surer technique than to simply skim the five choices and decidewhich of them seem like something in the passage. (Be sure to go back and check ‘em, ofcourse.) This technique works especially well when the choices are brief and theskimming is therefore easy. Employing this technique on Question 25 can pay off quickly.For one thing, you surely don’t need to check the passage to see that (D) is flat-out wrong— the colonies adopted many institutions from England, of course. One of those sharedinstitutions was a representative Parliament, so (B) cannot be the “pressure point”; in fact,so many things are described as having been shared that (C), too, should have jumped outas a major distortion of the text. “Uniquely English?” Not hardly. Unsure about (A), youshould probably recall that vocabulary comes up in Paragraph 1, and can quickly see therethat (A) is au contraire: Since “the colonials failed to observe that” they were using wordsdifferently (l. 6-7), that couldn’t have been a contemporary source of tension. This leaves(E), the right answer, support for which is found in line 10-12.Of course, if you simply saw the word “debates” in the question and found the word“debated” in line 9, the whole thing becomes moot: (E) is clearly supported right then andthere.

26. (C) We know that the issue of a constitution comes up in Paragraph 4 only, so youmight well want to skim the relevant paragraph before proceeding back to the answerchoices. How did the English attitude toward their Constitution differ from the colonists’attitude toward constitutions in general? Signals like “whereas” (l. 47) and “Thisdistinction in meaning” (l. 49) pinpoint the location, and probably anyone who’s ever takenCivics 101 was not surprised to learn that the English constitution is an unwritten set ofprecedents and conventions, while the colonists wanted the thing on paper. The former issummed up in (C), of course, and notice once again that right answers tend to useparaphrase to put a bit of a spin on the passage text, although you’re hardly going out ona limb by deciding that “a cumulative corpus of legislation and legal traditions” is a prettyclose equivalent of “the whole body of law and legal custom,” l. 45-46.

Page 19: Prep Test 9

KAPLAN LSAT PREP ________________________________________________________ LSAT Test IX Explained: Section I

© K A P L A N 17

That notwithstanding, many students erred and chose (A), because the rhetoric just cited,describing the English Constitution, certainly sounds lofty enough to justify (A).However, noting that that body of laws and customs dates back to the beginning of thekingdom is not to say that it was seen as the “legal foundation” of that kingdom. Indeed,the very fact that the English were willing to allow Parliament to change the Constitutionsuggests that they did not view it as such. (B), a limp distillation of (C), leaves out theentire legal issue. (As worded, (B) describes a copy of Emily Post’s Etiquette moreaccurately than any constitution.) Finally, that 18th-century Englishmen acknowledgedParliament’s right to change the English Constitution indicates that they didn’t regard theConstitution as “an unchangeable body of governmental powers” (E), although we can’t besure whether they had ever accepted that the King possessed the same right (D).

27. (E) Sometimes passages will come with two main idea questions associated withthem. When this is the case, one question will usually be more abstract (and, hence, moredifficult) than the other. Use your answer to the easier question to help you answer themore difficult one. Well, we’ve already seen that the big idea of the passage is that thecolonists and English gave different meanings to the same political terms, which, in turn,led them to assign very different roles to similar political institutions, causing somepolitical debate and disagreement between them. In more abstract terms, the passageexplains a dimension of the relationship between the American colonies and England inthe 17th and 18th centuries. So, choice (E) is the correct answer.(A) sounds like something carried over from poor Goody and his misinterpretation ofAthenian democracy in the previous passage. This passage discusses only the author’sviews, not those of other scholars; therefore, it can’t be said to expose a misunderstandingabout anything. Nor, for that matter, is there any hint that the author is trying to “settle adebate” about anything, so (C) can be just as quickly rejected. (B) is too narrow in scope:The brief glimpse we get of England’s treatment of the colonies (and the reason for it) in thelast paragraph can hardly be called the main focus of the passage. Finally, one wonders ifanyone chose (D) and, if so, whether that person ever really read (D). “Interpret the eventsleading up to independence”? American independence is maybe, maybe hinted at by lines2-3 , but that’s it. At best, one might say that this passage explains some of the attitudes thatlater contributed to the development of the American Constitution. But that was 1789, and(D) points us to 1776, and the author is interested in neither one.

Page 20: Prep Test 9

18 © K A P L A N

SECTION II:

LOGICAL REASONING

Page 21: Prep Test 9

KAPLAN LSAT PREP ________________________________________________________ LSAT Test IX Explained: Section II

© K A P L A N 19

1. (A) There are three pieces to this argument: evidence, counterevidence and conclusion.If that’s not obvious, look at the structural signals — the counterevidence is signaled by“however” and the conclusion by “therefore.” A quick skim down the choices, however,makes it clear that the overall structure won’t help us, since all the choices have the samebasic form. For parallel reasoning, an effective technique involves choosing a key piece ofthe argument (often the conclusion is best), then looking for a parallel among the answerchoices. The stimulus’ conclusion says that it’s not necessary to enact laws that addresshandgun-related crimes separately from other crimes. If we look down at the choices, (A)appears correct, since its conclusion — that it’s not necessary for physicians to emphasizethe danger of heart disease to overweight patients more than to other patients — carries theidea that it’s unnecessary to follow a given course based on a certain distinction. Using aclose paraphrase — like “unnecessary” for the original “no need to” — is a classictestmaker tactic in parallel reasoning questions. Others come close, but only (A)’sconclusion is parallel.(B)’s conclusion recommends that people who want to be healthy avoid swimming for fearof ear infections — the idea of a distinction between two groups is missing here as is theidea of an unnecessary recommendation. (C) clicks on the “not necessary” part, but, again,there’s no distinction between groups. (D) is about a group of people, those who eat richfoods, and makes a specific recommendation — but, without the idea that something isunnecessary, there’s no parallel. And (E) has a recommendation for certain people, butalso leaves out the “unnecessary” aspect. No need to remind you that you should ignorethe subject matter when looking for a parallel since, in this case, all the choices had similarsubject matter, and all were unrelated to the stimulus topic. Keep it in mind for futurequestions — ETS isn’t always this kind!

2. (D) What’s going on here? The author is assuming that because short children can’treach high shelves easily, they become short adults when they grow up. This is the crux ofthe argument, the flaw that breaks the camel’s back. If you miss the fatal flaw in themiddle of an argument, the conclusion may seem perfectly sensible. But if, as we haveadvised, you read the question stem first, you’ll know to keep your eyes open for it duringyour first read-through. So what’s wrong is that the author is taking a correlation: — shortchildren can’t reach high shelves easily and short children are likely to become short adults— and implying a causation: that the inability to reach high shelves causes the children togrow up short.As for (A), the author isn’t talking about Alice or Abdul, the short child, so there’s nomention of the characteristics of an individual member of a group. For (B) to be right, theauthor would have to assume that his conclusion (if short children are taught to reach highshelves, fewer of them will become short adults) were true before attempting to prove it —no dice. As with (A), (C) brings up the idea of an individual case, which doesn’t exist here.And (E) is way off, since the author doesn’t tell us that something doesn’t exist (like shortchildren?) because there’s no evidence that they exist. The confusion of correlation andcausation is an astonishingly frequent correct answer for logical flaw questions — keepan eye out for it.

3. (B) When tackling a main point question, especially one with a long stimulus like thisone, it’s often a good idea to paraphrase the argument . It should sound something likethis: It’s especially important to have balance when reporting on civil conflicts, eschewingbias and fact manipulation, but — this idea doesn’t justify concealing injustices in order tobe fair to both sides, since if everyone were to bend over backwards to seem impartial

Page 22: Prep Test 9

KAPLAN LSAT PREP ________________________________________________________ LSAT Test IX Explained: Section II

20 © K A P L A N

while ignoring real problems, the resulting picture would portray everyone equally, inviolation of reality or common sense. So the main point is that balanced reporting requiresthat one not only fairly report the views of each party, but also be sure not to ignore basicinjustices in order to seem impartial; and that’s what correct choice (B) says. Choice (A)erroneously picks up on the conclusion — if reporters ignored injustices, then balancedreporting would show that everyone has equal justification. This is a

detail, the author’s hypothesis of what could happen — not the main point. (C) is outsidethe scope, since the author doesn’t mention the idea of life experience making us aware ofspecific cases of conflict involving injustice. (D) just repeats the topic, while adding theidea of “common sense” gleaned from the last line, which gives us no idea of the point ofthe argument. And (E) might have been tempting, but it goes too far beyond the scope,since the author doesn’t mention the problem of subjectivity in reaching the ideal ofbalanced reporting. Watch out for choices that overemphasize details or go beyond thescope — these are the most common main point distracters.

4. (B) What’s the best way to weaken an argument? If you can’t recite it by now, take agood look: The best way to weaken an argument is either to undermine a centralassumption, or to provide an alternative explanation for the conclusion. This question isa classic example of the latter strategy. We’re told that deforestation in Melonia wassignificantly lower this year than in other years — and reliable proof is offered in the formof satellite photographs. The government of Melonia (land of melon?), having spent lots ofmoney to enforce antideforestation laws, is quick to take credit for the drop in forestdestruction.How can we weaken the government’s claim? Look for an alternative explanation, likechoice (B). If it’s true that the rainfall during burning season was abnormally heavy, thenit’s very possible that the drop in deforestation had nothing to do with the government — ithad to do with the rainfall instead. If this were the case, the government’s claim would besubstantially weakened. (A) tries to show that people protested the government’s efforts,but since we don’t even know that the government’s plan was effective, proof of oppositiondoesn’t help. (C)’s beyond the scope, since we have no idea if the number cited was a largeor a small number of violations or what effect this would have on the deforestation rate.(D) would weaken the idea that deforestation dropped significantly this year byundermining the proof, the photographs, but that’s not what we need to do — we need toweaken the government’s claim. (E)’s out as well, since the author says that the governmentspent millions of dollars enforcing the laws. We don’t care about how much money wasspent in other areas, since it won’t touch the government’s claim about the cause of thedrop in deforestation.

5. (D) A glance at the question stem before reading the stimulus tells us we’re to find theflaw in the advertisement, which is a common format for LSAT Logical Reasoningquestions. As always, it’s a good idea to compare the terms in the conclusion to those inthe evidence. Here, the ad’s conclusion is, in its inimitably cutesy style, that Northwoodssyrup is “tops in taste.” The evidence, flagrantly signaled by “and here’s the proof” (watchthose structural signals), is that seven out of ten shoppers with a preference preferredNorthwoods. Aha! The evidence, it turns out, doesn’t support the conclusion: theseshoppers never said why they preferred Northwoods, and they certainly never mentionedhow it tasted. So the ad is misleading because, as (D) has it, they could have preferred thesyrup for some reason unrelated to taste, say because it was cheaper.

Page 23: Prep Test 9

KAPLAN LSAT PREP ________________________________________________________ LSAT Test IX Explained: Section II

© K A P L A N 21

(A) actually makes the argument better by eliminating another potential weakness: that ahigh percentage of people asked had no preference for any brand. Look out for answerchoices that essentially do the opposite of what you were asked — they’re commondistracters. (B) is pretty sappy. (Sorry about that.) The “made the old-fashioned way”stuff was mentioned as an aside; the argument was that Northwoods is “tops in taste,” so(B) is irrelevant. (C) misses the point. Even if it were true that the market survey wasn’taccurate because it was based on too small a sampling, we’d still have to deal with thelarger problem that the shoppers never stated the reasons for their preference. Remember,you were asked for the strongest reason why the ad is potentially misleading. (E) commitsa classic scope error. The ad discusses only maple syrup, not any other type of syrup, so(E)’s point is irrelevant. Watch out for wrong answer choices that go beyond the scope —the testmakers love ’em.

6. (C) We need to resolve the apparent discrepancy between the poll’s prediction thatLandon would win and the fact that he was trounced by FDR. The author seems to believethat the survey sample of 10,000 was pretty representative of the U.S. population in termsof gender and place of residence, so bad sampling can’t be the problem. Or can it? (C)points out that a telephone survey, which this one was, eliminates everyone who can’t afforda phone, and that in 1936, phone ownership was less common than it is now. This weaknessin the survey sample may not have occurred to you when you first read the argument, butit should have hit you when you read (C). People without phones — i.e. the less-than-affluent — were beyond the pollsters’ reach, so (C) explains why their prediction flopped.(A) is irrelevant. How could the interviewers’ keeping their political affiliations tothemselves explain why the poll was inaccurate? If you thought (A) was meant to implythat the people conducting the poll were somehow biased in their predictions, well, there’sno evidence for this. Remember, answer choices that impugn arguers’ motives are usuallywrong. As for (B), it doesn’t matter that the poll covered only those qualified to vote, sinceit was meant to predict an election outcome, not determine which candidate everyone —including people ineligible to vote — preferred. (E), too, is irrelevant: knowing theinterviewees’ reasons for their preferences doesn’t solve the mystery of why the poll waswrong. Even if we knew why most of those polled planned to vote for Landon, it wouldn’texplain why he lost.(D) might have caught some of you who thought it implied that the polldisproportionately surveyed members of one party. But (D) doesn’t tell us that was thecase; it just says “no effort was made” to determine party affiliations. It’s possible that anequal number of members of both parties were surveyed, but that many people wereplanning on voting across party lines. Remember, the question stem asked for the bestexplanation of the five, and that’s (C).

7. (D) Whenever stimuli throw around percent increases or decreases, beware: Percentincreases and decreases are not the same as changes in the actual numbers or amount.Here the author signals the conclusion with “it is clear”: What is clear is that people arethrowing away just as much plastic or even more than ever before. We know that becausewaste management companies are reporting that an “ever-increasing” percentage of thewaste they handle is plastic. But percentages are deceiving; there may, for example, be ahigher percentage of plastic in the waste, but if the total amount of waste is decreasing,then the actual amount of plastic waste may have remained stable or even decreased.Choice (D) gets at this: Waste handlers are receiving less of other types of waste; thus there

Page 24: Prep Test 9

KAPLAN LSAT PREP ________________________________________________________ LSAT Test IX Explained: Section II

22 © K A P L A N

is less overall waste; thus the increase in percent of plastic waste does not necessarily meanthat people are throwing out as much as or more plastic waste.Choice (A) sets up an opposition that doesn’t exist in the stimulus. There’s no differencebetween plastic incinerated and that sent to a landfill; it’s all included in the plastic waste“handled” by the management companies. The point (B) raises — that some plastic isn’trecyclable, and therefore ends up in the garbage — would strengthen rather than weakenthis argument. (C) is irrelevant; whatever people do with plastic at home is not the issue.The question is whether they are putting more of it in the trash. Same thing with choice(D). The amount of plastic manufactured, or even an increase in the amount of plasticpackaging, isn’t necessarily connected to how that plastic ends up in the trash.

8. (D) In resolve-the-paradox questions, remember that both of the seeminglycontradictory pieces of the paradox are true, so you’re looking for an explanation thatallows them to peacefully co-exist. Here the givens are that between 1969 and 1986,stratospheric ozone, which absorbs a lot of ultraviolet, decreased. We’d expect acorresponding increase in ultraviolet, but instead we get a decrease. Why? Remember thatyou’re not trying to explain away the decreases; they’re a given. What you’re looking for issome new piece of information that explains why the cause — decreased ozone — didn’thave its predicted effect — increased ultraviolet. Could something else have absorbed theUV? That’s what correct choice (D) says.(A) tries to prove that there really was an increase in UV by pointing to increased skindisease: Since these diseases are caused by increased UV there must have been extra UVanyway, right? Wrong. The unexpected decrease in UV is a given and must be explained,not explained away. (B) tries to counter the stimulus by saying that in another part of theworld, a decrease in ozone did just what we expect it to do: raise the level of UV. But thatdoesn’t change the facts of what happened in North America. (C) explains why the ozonedecreased, not why the UV decreased. (E) may be true, but it’s outside the scope. Thefocus here is the situation in North America between 1969 and 1986, not on generalprinciples of ozone layer thickness.

9. (A) The stimulus for Questions 9 and 10 gives two pieces of information: the number ofairport ground collisions is going up because the number of flights is going up; and manyof the fatalities in those collisions occur because people can’t get to the emergency exits.The conclusion, not unreasonably, is that airlines should be obliged to clear away the seatskeeping passengers from reaching the exits. The choice that best supports that conclusion,and the answer to Question 9, is (A). Choice (A) provides an analogous situation — whenthey cleared a path to the exits in theaters, fatalities in theater fires went down. If it workedfor theaters, (A) suggests, why not for airplanes, too? (Note that they’re careful to specifythat these planes are on the ground.)You may have found Question 9 a bit tricky — you may have seen that (A) deals entirelywith theaters and thought, “this can’t be right, it’s beyond the scope,” and moved on. Forthis reason, Question 9 is a good illustration of why you should eliminate the wrongchoices. Sometimes you can get a difficult right answer simply by crossing off the choicesthat are less good. Suppose you passed over (A). (B) is obviously a wash-out — itsuggests a reason for not following the proposal in the stimulus, namely that it would bevery expensive. (C) is truly outside the scope: The fact that smoke detectors decrease firefatalities in public buildings does nothing to support the contention that airlines shouldprovide better access to emergency exits. (D)’s distinction between small and large planesdoesn’t help either; if anything, because it seems likely that access to exits is better in small

Page 25: Prep Test 9

KAPLAN LSAT PREP ________________________________________________________ LSAT Test IX Explained: Section II

© K A P L A N 23

planes than in large planes, (D) undercuts the claim that restricted access to emergencyexits is the problem. (E) gives another safety tip — seatbelts protect passengers — but thathardly supports the conclusion that airlines should clear seats away for easier access toemergency exits. So it’s back to (A); it may not provide very strong support, but it’s theonly choice that provides any support at all.

10. (B) We’re faced with a peculiar question stem here: We’re looking for a proposal thatwould work together with the passage’s proposal and improve its prospects for reducingfatalities. The correct answer should help ensure that clearing away seats to provide betterexit access actually will decrease fatalities. (B) does the job because it deals with the otherfactor mentioned in the stimulus — that the increase in collisions was caused by an increasein flights. If (B) weren’t to be implemented, if airlines did increase the number of flights,then there would be a danger that the number of collisions would increase as well, andthus that the total number of fatalities would not decrease, even though each individualcollision might be safer. (B) eliminates this possible reason for an increase in collisions,and gives the stimulus proposal a better chance to work.Of the other choices, only (A) is tempting — it says that any new planes bought mustalready meet the proposal’s safety requirements. However, (A) doesn’t really add anythingto the passage’s proposal. According to that proposal, whatever type of airplane theairlines buy, they’ll have to clear access to the exits before using the planes — it might savetime and money to buy airplanes in which this was already done, but it wouldn’t help savelives. Also, we don’t know how many (if any) new airplanes the airlines intend to buy — ifthey buy many, they run the risk that (B) tries to avoid, that of overcrowding. (A) mayhave been tempting, but you should have rejected it after reading (B) — a good illustrationof why you should read all the answer choices carefully in Logical Reasoning.The other choices are wide misses. (C) and (E) talk about check-in procedures and ticketprices, which have nothing to do with passenger safety. (D) talks about “securityprecautions,” a little nearer the mark, but it only vaguely speaks of “defining” them moreclearly, not of making them more effective; in any case, security precautions weren’tmentioned in the passage. Even if you couldn’t decide between (A) and (B), you shouldhave found it easy to eliminate the others — if worst comes to worst be ready to eliminateobviously wrong choices to improve your odds of guessing correctly.

11. (E) This question is another that exemplifies why reading the question stem first is agood strategy. You’re asked for the method of argument, and if you know that from theoutset, it’s easy to pay attention to the stimulus’ structure as you read, keeping a “mentalmap” as you do with Reading Comp passages. As (E) says, the argument presentsevidence that a past phenomenon — I guess dinosaurs can be considered a “phenomenon”— is more similar to one rather than the other of two present-day phenomena. Dinos, theargument claims, are more like present-day warm-blooded animals (such as birds) thanthey are like present-day cold-blooded animals (such as reptiles). And the evidencecompares the dinos of yesteryear with birds and reptiles of today. (Perhaps if you recalledthis bird vs. reptile thing from Jurassic Park it was a little easier to understand.)Contrary to (A), the argument doesn’t discuss the information that the opposing position— that dinosaurs were cold-blooded — is based on. It makes its case by discussing theevidence for its own side, not by tearing apart the opposing side’s evidence. As for (B), theargument doesn’t state a “general principle,” then apply it to a particular case. Rather, itbuilds up a claim that dinosaurs may have been warm-blooded (which can’t really becalled a “general principle”) based on fossil evidence — so it really moves from the

Page 26: Prep Test 9

KAPLAN LSAT PREP ________________________________________________________ LSAT Test IX Explained: Section II

24 © K A P L A N

specific to the general, not the other way around. (C) has everything twisted: Theargument uses historical evidence (about dinos) as well as contemporary evidence (aboutpresent-day birds and reptiles) to make a claim about the past (that dinos were warm-blooded); it doesn’t dismiss a claim about the present. (D)’s gobbledygook would describean argument something like this: All warm-blooded animals have hollow bones. Dinoshad hollow bones. Therefore, dinos were warm-blooded. But the argument doesn’t dothis: It doesn’t discuss one “certain property,” but builds its case from several examples oftraits shared by dinos and present-day warm-blooded creatures. And the language is alsomore qualified than (D) would have it: It says that some, not all, dinos had hollow bones;some dinos had a highly arched mouth roof; many had a growth rate typical of warm-blooded animals, etc. Don’t let yourself get confused by abstract language — take itslowly and calmly, and you can cut through the jargon.

12. (A) As we said back at Question 4, one great way to weaken an argument is toundermine its central assumption. And this author is assuming that these “geneticallydetermined abnormalities” that afflict purebred dogs are a serious problem. Without thatassumption, there’d be no reason for him to argue that potential dog owners should choosenonpurebred dogs in order to avoid having to pay for the expensive surgery needed tocorrect these abnormalities. If (A) is true, if the abnormalities don’t seriously harm the dog,then the surgery is an unnecessary expense and potential dog owners needn’t worry abouthaving to avoid it.(B)’s a typical wrong answer choice — it falls outside the scope of the argument. Theargument only addresses genetically determined diseases, so evidence aboutnongenetically determined diseases won’t weaken it. (C) and (D) provide us with new, andirrelevant, information. Neither the shorter life span of purebred dogs in (C) nor theirhigher purchase price in (D) speaks to the central issue, which is avoiding the costlysurgery the author claims is needed to correct genetic abnormalities in purebreds. At best,(C) and (D) can only strengthen the author’s argument by presenting even more reasons toeschew purebreds: Not only are they likely to need surgery, but they tend to die soonerand they’re more expensive to buy. Finally, (E) tries to weaken the case for buyingnonpurebreds by pointing out that dogs without genetically determined abnormalities canstill have puppies with such abnormalities. Again, irrelevant. The author only claimedthat dog owners can avoid costly surgery by buying nonpurebreds. He never saidanything about what may or may not happen to the dogs’ offspring, so (E)’s informationdoesn’t weaken his argument.

13. (D) Question 13 is a disguised formal logic question. It’s hard to follow, and it helps alot if you read the question stem first: That way you know enough to read the stimuluslooking for what deductions you can make about the press. The heart of the argument is asfollows: If the press were not a profit-making institution, it would be subsidized andsubject to outside control — there’d be no other alternative. Furthermore, it’s easy to getsubsidies for propaganda, but it’s impossible to get subsidies for honest journalism. We caninfer from this last statement that if the press produces honest journalism, it isn’tsubsidized. Combine that with the statement that the only possible alternatives are asubsidized press and a profit-making press. What’s the conclusion? If the press produceshonest journalism (and therefore is not subsidized), then it must be profit-making — andthat’s choice (D). To check such an answer, consider its contrapositive. “If it’s not gonnamake a profit (i.e. if it’s subsidized), then it’s not gonna be honest.” Just what the authorsays! An inference, remember, is a statement that is virtually certainly to be true based

Page 27: Prep Test 9

KAPLAN LSAT PREP ________________________________________________________ LSAT Test IX Explained: Section II

© K A P L A N 25

on the stimulus. And in a large number of cases on the LSAT, the correct answer will bethe contrapositive of something the stimulus author stated.You might have been able to pick choice (D) by quickly scanning the answer choices afterunpacking the stimulus. On the other hand, sometimes the only way is to slog througheach of the choices one by one. (A) can’t be inferred; the argument says that if the press isnot profit-making, then it will be subsidized and subject to outside control. This does notmean that if the press isn’t subsidized it will of necessity be free of outside control — aprofit-making, outside-controlled press is still a possibility. (B) makes a similar mistake —even if it’s true that the press that doesn’t produce propaganda doesn’t get subsidies, thatdoesn’t imply that the press that doesn’t get subsidies doesn’t produce propaganda. (C) iscompletely confused — the argument says that if the press isn’t subsidized it must be aprofit-making institution. Finally, we showed that, as (D) says, the passage implies that ifthe press produces honest journalism, then it must be profit-making — (E) commits theerror of turning this around (affirming the consequent, don’tcha know) and concludingthat if the press is profit-making, it must be honest. We most definitely can’t conclude that!

14. (C) One stimulus, two questions — more bang for your buck. With dialoguequestions such as this one, it’s often a good idea to read the shorter portion, here Maria’sstatement, first. You can thus stay ahead of the situation by making clever and solidinferences about what the first person must have said. Then, reading the longer speechbecomes a matter of confirming those inferences — which is much easier to do than toread a long peroration from scratch! And anyhow, since you read the question stem aheadof time, you knew you were supposed to critique Lucien’s logic, which is easier to do ifyou know what Maria’s objection is. Well, Maria tells us that many homeless people haveregular jobs and therefore don’t lack will or ability. Since she prefaces this remark with“on the contrary,” we can infer that Lucien believes that the homeless don’t have jobs andthat they do lack “will or ability,” presumably the will or ability to work.As for Lucien’s argument, he begins by calling “absurd” the claim that more low-incomehousing is needed because there’s not enough housing available for the homeless. Then hestates his evidence: Many apartments in his building and those of his colleagues arevacant, so the problem clearly isn’t availability. Therefore, it must be that the homelesscan’t or won’t work to pay the rent. This last part, remember, is what Maria disputed.Well, who says the homeless, even if they worked, could pay the rents for the apartmentshe mentions? This is (C)’s criticism: Lucien completely misses the advocates’ point, whichwas that there’s not enough affordable housing. “There are lots of apartments where I live,”he reasons idiotically, “so there can’t be a housing shortage.” But “plenty of housing”doesn’t mean plenty of affordable, low-income housing.We won’t dwell on the wrong choices. Contrary to (A), Lucien does justify why he calls theadvocates’ argument absurd. His justification is rather absurd, but it’s there. As for (B), ifthe “controlled scientific studies” it mentions refer to Maria’s evidence, then it would onlywork if we knew Lucien’s response to her objection. True, he uses anecdotal evidence, butwe don’t know that he considers it as strong as Maria’s evidence, since we don’t know hisreply. And if (B) doesn’t refer to Maria’s evidence, then it’s just irrelevant; either way, (B)fails. Lucien never claimed that every building has vacant apartments in it, so (D)’s nocriticism of his argument. And (E) misses the point as well: contrary to its claim, theadvocates never raised the issue of who would pay for low-income housing.

Page 28: Prep Test 9

KAPLAN LSAT PREP ________________________________________________________ LSAT Test IX Explained: Section II

26 © K A P L A N

15. (B) Now we have to describe Maria’s method of argument; she, you’ll recall, pointedout that many homeless people work and therefore don’t lack the will or ability to do so.As (B) says, her comment demonstrates that a presupposition of Lucien’s argument is false.A presupposition is a just a fancy word for an assumption, and in order for Lucien toconclude that homelessness is caused by people’s unwillingness or inability to work, hemust assume, of course, that the homeless don’t work. After all, if he assumed that theydid work, he’d have to conclude that homelessness was caused by something else — say,by a shortage of housing that the homeless could afford.I think it’s pretty clear that Maria’s response isn’t intended to challenge the accuracy ofLucien’s personal experiences, as (A) says. Although his use of anecdotal evidence isanother possible weakness in his argument, she doesn’t address it. And her evidencedoesn’t bring up the issue of his motives, so (C) is out. Regarding (D), Maria doesn’t pointout that Lucien’s evidence — his claim that there are many vacant apartments — shouldlead to a different conclusion; in fact, she never deals with his evidence at all. And as for(E), it’s hard to see how her evidence that many homeless have jobs could be interpreted asan alternative explanation for his evidence about vacant apartments. Again, she doesn’taddress his evidence; she undermines his assumption with her own evidence.

16. (A) It’s a short but dense stimulus for Question 16. You may want to break densestimuli down into smaller blocks, and paraphrase confusing words. What’s the authorsaying? Some people get their moral standards from governmental codes of law. Thesepeople cannot imagine that something that is legal could be immoral. In other words, forthese people, if something is legal, then it must be moral. This is one of those formalarguments in casual clothing, and if you remember your if-then statements, this should beno sweat. If not, go back and learn them!So we need to find a statement that is logically inconsistent with the stimulus statement —something that, if people held both views at the same time, would give them aninconsistent outlook. If something is legal, then it is moral, they say. What’s thecontrapositive of this statement? If something is not moral, then it is not legal. Forsomeone to hold logically consistent views, they must believe the contrapositive of theirbeliefs as well, right? Well, choice (A) is inconsistent with this belief, and therefore correct,because it implies that it’s not necessarily true that if an act is immoral, it is illegal, sincelaw doesn’t cover all immoral circumstances.As for the other choices, (B) says that if something is not legal, it’s not moral. While that’snot necessarily true in this context, it’s also not inconsistent, so it can’t be the right answer.(C) says that government officials do illegal things sometimes; well, government officialsare beyond the realm of relevance. (D) talks about laws as the moral consensus of a society,but the stimulus just tells us that people take moral cues from the law, not that laws aremade based on people’s moral codes. And (E) is way out there — it’s beyond the scopesince the economic implications of government regulations mean nothing to us. Withcomplex formal logic, keep it as simple as possible. Break down the language, and take itpiece by piece.

17. (B) If this one reminded you of a logic game, so much the better: Your LG skills serveyou well here. First, note that the question asks for the one thing that cannot be true. Next,draw out the information the stimulus gives — and doesn’t give. There are two sets ofinstruments; the same procedures are done to each; and the procedures yield similarresults. It takes more energy to sterilize a set of these instruments than it does tomanufacture them. But we don’t know how many instruments make up each set, nor do

Page 29: Prep Test 9

KAPLAN LSAT PREP ________________________________________________________ LSAT Test IX Explained: Section II

© K A P L A N 27

we know exactly how much energy it takes to manufacture either set. It could take more tomake nylon tools, or it could take more to make stainless steel, or it could take equalamounts of energy for both. We just don’t know. Unfortunately there’s no way to predictthe answer here. It’s just a matter of slogging through the choices until you find the rightone.(A) could be true. If the same amount of energy was required to manufacture both thenylon and the steel instruments, then sterilizing the nylon instruments would have beenmore energy consuming than sterilizing the steel. (B), however, can’t be true, and yes, ittakes a little math-thinking, but that’s life. (You could have skipped this choice andchecked out the others; if all of them can be true, then (B) must be correct whether or notyou understand why.) If 50 complete sterilizations of the nylon instruments took 3.4 timesthe energy to make that set, then they should be able to do roughly 15 sterilizations withthe same energy as manufacture. One complete sterilization, then, must have taken far lessthan the energy of manufacture. (C) could be true; as we noted above, we don’t know howmany types of instrument appear in each kit, so there could be more nylon instrumentsthan steel. Since both kits were sterilized an equal number of times, more nyloninstruments could have been sterilized. (D) absolutely could be true, since we don’t knowhow much energy was required to produce either set. (E) is also quite possible. Ifmaterials are cheap, and energy is very expensive, then using far more energy to sterilizethan manufacture steel instruments could make sterilizing them more expensive thanmaking them in the first place.

18. (E) This is a fairly common question-type on the LSAT, one I hope you findinteresting: a “principle question.” Here you’re asked to choose which action could bestbe justified by the principle to which the group leader appeals. You can think of suchprinciple questions as very abstract parallel logic questions — you want the concretesituation that best fits the “blueprint” laid out in the principle. And as in parallel logicquestions, you can reject any choice that’s missing any of the key features of the model.Here the principle is stated very clearly: If citizens make a good-faith effort to complywith a law, but are prevented from doing so by government inaction, then they need notrefrain from actions that fail to comply with the law. The thing to do is to look through theanswer choices, rejecting those that aren’t fully covered by this principle.(A) only talks about a chemical company’s own report; there’s no mention of the law, or ofgovernment action or inaction — the situation clearly has nothing to do with the principle.In (B), the city resident is faced with government inaction, but elects to comply with the lawby forgoing the construction; the principle concerns those who fail to comply. In (C), thecar owners are not making a “good-faith effort to comply with the law,” frustrated bygovernment inaction; instead, they’ve found that they can get away with breaking the lawwithout being prosecuted. But the government inaction in this choice concerns punishingthose who break the law. There’s no suggestion that government inaction has made itdifficult to comply with the law, as was the case in the stimulus — people could getinsurance, they just don’t want it. (D) may be a bit tempting, but read it carefully — thedeveloper doesn’t fail to comply with the law, he has a valid permit. So we’re left with (E),and sure enough, it’s a perfect fit. Here’s a physician who made an effort to comply withthe law (he applied for a license); it was a good faith effort (he met all the qualifications);but he was thwarted by government inaction, so he went ahead and broke the law. Perfect!

Page 30: Prep Test 9

KAPLAN LSAT PREP ________________________________________________________ LSAT Test IX Explained: Section II

28 © K A P L A N

19. (D) and 20. (A) Reading both stems (19 and 20) before reading the stimulus they sharemakes your work on this pair much easier: In both cases, the questions concern good oldnuts-and-bolts structure of the argument, so you know you’re reading to identify theconclusion and assumptions. In fact, quite often it’s smarter to try the second of tworelated questions first; the testmakers often set it up that way, to reward those who areclever and startegic. That’s certainly true here, because when you’re finished withQuestion 20 you have identified a part of the argument’s structure, which should help youanswer Question 19.What does the author of this argument want to convince someone else to believe? That auniversity should not have the right to patent faculty inventions. The rest of the argumentexplains why that is so: in other words, the remainder of the argument is evidence, and“the claim” that a university should not have the right to patent faculty inventions is theconclusion. That makes (A) correct for Question 20. Very simple, very straightforward.Isn’t that cool?It’s worth noting that of Question 20’s wrong choices, only (C) is a concise description of afamiliar “role” in Logical Reasoning arguments as you know them: an assumption. Butwhat’s an “explicit assumption”? Assumptions by definition are the unstated linksbetween evidence and conclusion. They aren’t explicit at all, and the given piece of theargument in the question stem isn’t an assumption. Choice (B) doesn’t make much senseeither, but is easy to eliminate when you realize that whatever it means, it’s not talkingabout the conclusion. It’s talking about some piece of the argument that leads to aconclusion, but we need a clear choice that says “it’s the conclusion” (and we got one). (D)describes what in Reading Comp we’d call a detail. In Logical Reasoning there aren’t manydetails, since the prose is so short and tightly written. Even if you couldn’t otherwiseidentify the piece of the argument in question, this choice isn’t a good bet as a guess. We’renot sure what the test writers were getting at with (E), but you can reason your way pastthis one as you did with (B): (E) describes something that argues against the conclusion.We’re looking for the conclusion.This work should make answering Question 19 easier. You’ve already identified theconclusion and the evidence. Why shouldn’t universities have the right to patent facultyinventions? Because the right might be abused; the university might serve its own interestsinstead of the public interest by suppressing new discoveries and inventions until it canobtain a patent, and a university should promote knowledge of new inventions, not hidethem for any reason. To say that anyone should not be given a right because they mightabuse it is to assume that at least some people will abuse it. That’s the heart of what correctchoice (D) says.(A) may be something this author takes for granted, but it’s not relevant to this argument.The argument doesn’t compare the rights of universities to the rights of any otherinstitution, for the simple reason that the scope of this argument doesn’t go beyonduniversities. The fact that the university is responsible for the free flow of informationdoesn’t suggest that no other institution shares that responsibility. (B) may be temptinginitially, if you’re thinking that without the profit motive, a university would be unlikelyto abuse patent rights; but do you see that even with that rationale, the real link betweenevidence and conclusion is still the possibility that some universities will abuse theirpatent rights? (C) focuses on “publication,” a specificity the stimulus argument lacks.How information is disseminated is simply not an issue here. (E) introduces anotherirrelevant issue: exactly who will patent inventions if the university does not. The scopehere is strictly limited to the rights and responsibilities of universities; the argumentdoesn’t address the results of anyone else’s patenting inventions.

Page 31: Prep Test 9

KAPLAN LSAT PREP ________________________________________________________ LSAT Test IX Explained: Section II

© K A P L A N 29

21. (D) The author’s conclusion only works if (D) is true: if there’s no third language fromwhich these two languages got the word “dog.” If you don’t see that, try the denial test —the best way of seeing whether something is in fact being assumed. If (D) were false, whateffect would it have on the argument? If there were some third language from whichEnglish and Mbarbaram borrowed the word “dog,” then that would blow apart theauthor’s conclusion that these similarities can arise without language borrowing havingoccurred. In this case, language borrowing would have occurred, but indirectly, via athird language. When the denial of a choice counters the argument, then that choice issomething that the author has assumed.The author needn’t assume that the only word shared by these languages is “dog,” so (A) isout. Try the denial test again: If she assumed the existence of other words like “dog” thatsupposedly weren’t borrowed, then these words would just be more evidence for herconclusion. And if she assumed the existence of words that were borrowed, they’d beirrelevant, because her argument only concerns allegedly non-borrowed words. Contraryto (B), the author needn’t assume that any other languages possess the word “dog.”Again,the denial test: If no other languages besides English and Mbarbaram used the word“dog,” that wouldn’t weaken her argument that the similarity occurred without borrowing— if anything, it’d strengthen it. So (B)’s not being assumed. (C) is a possible, but notnecessary, assumption. The author could assume that word sharing between two languagesusually occurs when the languages are related. But this isn’t necessary to her argument,which, if you read carefully, concerns what sometimes happens, not what usually happens.If you don’t see that, one more time, try the denial test: If she assumed that two languagesthat share a word usually aren’t related (which is kind of counterintuitive), it wouldn’taffect her argument either way. Again, she’s arguing that word sharing can sometimesoccur without language relatedness or word borrowing. Assumptions about whenlanguages are and are not related are beside the point. Finally, (E)’s conclusion impliesthat the only way unrelated languages can share words is by borrowing them from oneanother. But if the author assumed this, her argument would be blasted to kingdom come:Her whole point is that it’s possible for such languages to share words without borrowinghaving occurred.

22. (D) Here’s something a lot of us like to do in our spare time: critique the logic ofpoliticians. Moreover, this is another of those common LSAT Logical Reasoningquestions that play off a confusion between percents and numbers. The politician saysthat since his party took office, the number of unemployed people increased by under 20percent. He then compares this to the record of the opposition party, during whoseadministration the number of unemployed people increased by over 20 percent. Heconcludes that there are now fewer people unemployed, thanks to his party’s leadership.Well, that’s tommyrot. He just finished saying that more people are unemployed; it’s justthat the rate of increase in unemployment is lower now than it was under the previousparty’s rule. As (D) says, his evidence actually supports the opposite of his conclusion: Itsupports the conclusion that there are more people unemployed now, not fewer. (D) winsby a landslide, and let’s kick this politician out of office.We can infer from the politician’s last sentence that the opposition claims that the numberof unemployed people is now higher due to the leadership of the politician’s party. Thus,contrary to (A), he does specify the opposition’s claims. (B)’s not a valid criticism. Theflaw in the politician’s argument isn’t that he hasn’t shown a uniform decline inunemployment; it’s that he hasn’t shown any such decline. After all, he’s only arguing that

Page 32: Prep Test 9

KAPLAN LSAT PREP ________________________________________________________ LSAT Test IX Explained: Section II

30 © K A P L A N

there are fewer people unemployed overall; he’s not arguing that unemployment declinedeverywhere in the city by the same rate.The issue of seasonal fluctuations in unemployment, raised by (C), is irrelevant. Thepolitician’s not concerned with the reasons for unemployment; he’s just arguing that fewerpeople are unemployed now, for whatever reason. As for (E), by talking about the“increase in the number of people employed,” it accepts as valid the politician’sconclusion that there are now fewer people unemployed. As we saw, that was the wholeproblem with his argument. So (E) makes the same error of reasoning as the politiciandoes.

23. (A) It’s another formal argument in casual clothing for Question 23, and there’s amissing assumption for us to track down. What’s our poor farmer saying? Everyone iseither rich or poor. Also, everyone is either honest or dishonest. All poor farmers arehonest. Therefore, all rich farmers are dishonest. Wait a minute — we’ve accounted for thefact that every farmer who is poor is also honest. But remember that just because all poorfarmers are honest, it’s doesn’t necessarily mean that all honest farmers are poor. It’spossible that there could be rich farmers who are honest. So, by concluding that all richfarmers are dishonest, the author is assuming that every farmer who is honest is also poor.Don’t believe me? Here it is again: When in doubt,try the denial test to check anassumption. If you negate the choice in question, and the argument still holds, then thatchoice is not correct. However, if the argument falls apart, then it’s the answer we seek.Well, if the opposite of (A) is true, if not every honest farmer is poor, and all people mustbe either poor or rich, then some honest farmers must be rich. That contradicts the author’sstated conclusion that all rich farmers are dishonest, so (A) must be correct.We needn’t assume (B), that all honest people are farmers, since the author’s conclusionpertains only to farmers, not to people in general. (C) also goes too far, since we needn’tassume that all dishonest people are rich farmers in order to conclude that all rich farmersare dishonest. (D) brings in other people again, and we’re just concerned with farmers —all poor farmers are honest, but that doesn’t mean that all poor people are honest. Alwayskeep an eye on the scope of the subject matter. And (E) does just the opposite, claimingthat all poor people are farmers — we needn’t assume that either.

Page 33: Prep Test 9

KAPLAN LSAT PREP ________________________________________________________ LSAT Test IX Explained: Section II

© K A P L A N 31

24. (D) We need to parallel the novelist’s reasoning — you can breeze through thejournalist’s comment, just to set the stage, and then focus exclusively on the second half ofthe argument. As we’ve been doing with other parallel reasoning questions, instead ofassimilating the argument as a whole, let’s just focus on the most important portion, theconclusion, first. The novelist claims that what she wants to communicate can only comeacross in the form of a novel, so she can’t summarize what it’s about, since the medium ofthe novel is essential to what’s being communicated. The choice we’re looking for needs tohave this idea of something that can only be conveyed in one way, making a substitutetotally unacceptable. (D) is perfect, because here the author explains that a threedimensional landscape can’t be captured with a traditional camera. No other choice hasthat same sense of the impossibility of some action fully conveying an experience (andwhile (D) and the original shouldn’t be compared in terms of topic, they do both share aconcern with the execution of art, which strengthens the parallel.)As for the others, (A) gives a specific definition of a blueprint, then explains why a givendrawing can be considered to be a blueprint. (B), like (A), defines something, a pressrelease in this case, and explains why another object doesn’t fit into the same category. (C)contrasts actual travel with travelogs, and concludes that the two are not the same. Yet it’smissing the idea that there’s something that cannot be truly conveyed through any mediumother than the original — there’s no claim that a travelog can’t duplicate the true feel oftravel. And (E) is rather bizarre, since the function of a menu before and after a meal is farbeyond the scope of the stimulus argument’s structure.

25. (A) Searching for the author’s assumption in a long and dense argument like this onecan be tough — try underlining key evidence and conclusion if you’re getting lost in densetext. What’s happening here? Well, medical findings aren’t released to the public untilafter they’ve been published in a journal that has given them peer review. People claimthat this delays access to potentially lifesaving information. Yet peer review is the onlyway to keep harmful information from reaching an unequipped public. Therefore,waiting until a journal has done peer review and has published the findings is the onlyway to protect the public from uninvestigated research.So what’s the author assuming? Well, she tells us that peer review is the only way that theinformation can be okayed, then concludes that the public has to wait until a medicaljournal has published the findings. But what if there’s some other way that findings canundergo peer review? If this were true, then the author’s conclusion would be invalidated.Yes, that’s the denial test, presented in a sneaky way. So (A) must be a necessaryassumption. (B)’s way off, since we have no reason to believe that there aren’t plenty ofpeople who would evaluate medical research findings who don’t happen to be on medicalreview panels. (C) tries to lead you down an erroneous road — it’s not that the public can’tread the journals once they’re published, it’s that the public shouldn’t be presented withpotentially harmful information that hasn’t been adequately reviewed. As for (D), we haveno reason to assume that all findings are subjected to peer review — maybe some never getthat far, and are halted before they reach that stage. And (E) is way beyond the scope, sincewe needn’t assume anything about the partiality of peer review panels.

Page 34: Prep Test 9

32 © K A P L A N

SECTION III:

LOGIC GAMES

Page 35: Prep Test 9

KAPLAN LSAT PREP _______________________________________________________ LSAT Test IX Explained: Section III

© K A P L A N 33

This Logic Games section was certainly interesting, if not especially difficult. Kaplan'sTeam LSAT found noteworthy the total lack of any sequencing element to the games in thesection — the first time that that’s ever been the case. Does that mean that the Kaplanstudents who took this exam were left out in the cold? Hardly. Making up for themysterious (and most likely temporary) disappearance of sequencing games was apreponderance of grouping games: No less than three out of the four games fell under thismantle. Games 1 and 4, “Three Corsages” and “Hospitals, Jails, and Universities,”respectively, are grouping games of distribution, although the latter has a strong spatialarrangement element, complete with artwork. The former is fairly standard. (Again, whileit's impossible to predict whether the absence of sequencing was just an anomaly or thebeginning of a trend, history certainly suggests that sequencing's absence is indeedtemporary; it has always been the #1 game type up to now.)The setup of the other grouping game, Game 2 (“Diplomat’s Dinner”), was virtuallytextbook — selecting four people from a group of seven. Even the rules looked to bestraight out of our lessons, Home Study, and supps. So it was surprising when someOctober test takers reported having difficulty with this one. Now that we've seen the game,we figure that the snag was the fact that three of the six questions supply no additionalinformation, which really puts the burden on the test-taker to do a lot of analysis givenvery little direction.Of course, as will be discussed later in this chapter, the test-taker who took our advice toheart — who spent quality time up front analyzing the situation, making deductions, andplotting out as much as possible — surely had much less trouble with the game. True,some games allow you to fudge things in the beginning and feel your way as the questionsprogress; you can pick up what you need to know in order to answer a few questionscorrectly. But this game allowed no such luxury: Those who skipped the pre-thinkingstage got killed, while those who took the time to work through the possibilities wererewarded.Finally, the only non-grouping game on the test should have come as no surprise to youKaplan students — “Dance Recital,” Game 3, is a straightforward matching game. It mayappear to have a sequencing element (the three dances in a row), but look again — the issueof who dances before and after whom is never considered.As you know, we feel it’s imperative that you preview the section and set up an overallstrategy before jumping into any one game. It seems as if this time around there wasvirtually nowhere to turn without bumping into grouping, but considering Kaplan’swealth of grouping materials, including a full Topical Chapter in the Logic Games HomeStudy Book, most Kaplan students who took this exam ought to have found the LogicGames section both familiar and manageable. If you had trouble anywhere along the line,that’s what we’re here for: We’ll try to show you how to avoid similiar trouble later on.

Page 36: Prep Test 9

KAPLAN LSAT PREP _______________________________________________________ LSAT Test IX Explained: Section III

34 © K A P L A N

Questions 1 - 7

Game 1 really bombards you with numbers — one florist, making three corsages, from fourtypes of flowers, each individual corsage containing three flowers. Later, we’ll see thatsome of the rules are number oriented as well. Sound confusing? Well, try not to let all ofthe number information throw you; this is still a typical game type that you should befamiliar with from the Logic Games Lesson One and your Kaplan practice materials —namely, a grouping game of distribution. How can we tell that it’s a grouping game ofdistribution? Well,when the action of a game appears complex, always try to break itdown into its base elements. The lowest common denominator here is the types of flowers— gardenias, orchids, roses, and violets — that make up the corsages. Each corsageconsists of three flowers, so with three corsages altogether, there must be exactly nineflowers. Our job will be to place, or distribute, the letters G, O, R, and V (short for thetypes of flowers) into those nine slots or spaces — hence, a grouping game of distribution.Now, simply knowing the game’s formal name, by itself, will help you little. What willhelp you greatly, is when all of your practice with this game type “kicks in” and yourealize that it’s not an alien entity, but something that you’ve done before and that’scertainly manageable now. If you preview the entire game, which is the first step of theFive Step approach, you’ll see that the questions are very standard: One asks for anacceptable selection, while another common grouping question, finding a maximum forone of the entities, appears more than once. Furthermore, notice that most of the questionstems contain hypotheticals, and that’s good: The more they give you to work with, thebetter.As for sketching this one on your page, I think that a simple 1, 2, and 3, each with threedashes underneath, will suffice. Now comes the fun part, working through the rules tofigure out just what constraints will govern our distribution of flowers. And don’t thinkthat there are only three rules here — there are only three indented rules. Be on the lookoutfor pieces of information in any game’s introduction that qualify as rules: Here, we'retold to include at least one flower of each type; in other words, we can never entirely leaveout any letter (G, O, R, or V). Well, that's something you may just recall, or if necessaryyou can jot down a note off to the side, something along the lines of “USE ALL." However,the second piece of information, about roses and orchids, is a genuine rule. Don’t casuallyglance over things in the game’s introductory paragraph in your haste to get to the rules.You may very well miss an important rule that the testmakers decidedto separate fromthe indented ones.Okay, so let’s look at that number rule. Whenever you’re given a rule involving numbers,it's wise to take the time to think it through; don’t simply write it down and forget aboutit, but take a few seconds to work out the possibilities. There are nine flowers total.There must be at least twice as many roses as orchids. Furthermore, we know there mustbe at least one gardenia and one violet as well. So, the operative question is, given thesecircumstances, "How can this be done?" This is the concept that the weak logic gamesstudent is most likely to ignore. Don’t be satisfied merely to acknowledge a rule orstatement — ask yourself: How can this be accomplished? Well, here’s how: If there’s oneorchid, then there must be at least two roses, and there'll be plenty of room to get gardeniasand violets in also. If there are two orchids, then we'll need at least four roses, totaling sixof the nine flowers; that'll still leave room for the other two types. But that’s it! We can't goto three orchids because that would mandate six roses, which'd be nine right there withouta single gardenia or violet. Conclusion? There will be either one or two orchids in the

Page 37: Prep Test 9

KAPLAN LSAT PREP _______________________________________________________ LSAT Test IX Explained: Section III

© K A P L A N 35

corsages, and I indicated that next to my sketch — “ONE OR TWO Os.” You need jotdown no more than that, provided you remember the orchid/rose requirement.Okay, as for the indented rules, Rules 2 and 3 give us the most concrete information, so let’sbegin there. In my master sketch, in one of the dashes for corsage 2, I wrote in the letter R,taking care of Rule 2; and I placed the letter G in one of the dashes of corsage 3, asmandated in Rule 3. That rule also forbids us from placing any orchids in corsage 3, so Iwrote “NO O” next to corsage 3. Rule 1 is less specific, but important nonetheless. Youmay have gotten confused by the wording, andwhenever that happens it's wise toclear up the confusion by using an example to illustratethe meaning of the rule. If that example doesn’t work, then that tells you that you're stillmisunderstanding the rule. Usually, finding an example that works helps align yourthinking with that of the testmakers. Here, for example, it might strike you as odd thatcorsage 1 is required by Rule 1 to contain exactly two types of flowers. At first, I thought,“wait a minute, each corsage must have three flowers, what’s the deal?” So I tried anexample — one gardenia and one rose. That’s no good, we need three flowers. But if I addanother gardenia, then I have three flowers, good, and yes, only two types — gardenias androses. Only after choosing a simple example did I see the difference between two flowersand two types of flowers. It also forced me to think about what corsage 1 couldn’t look like:A rose, a gardenia, and a violet, or any other combination of three types, will not do. Thepoint is, once you think things through, it really doesn’t matter what you write on thepage — you'll probably recall the rule so long as the concept is stuck in your head. So ifyou write something like “JUST 2 TYPES” next to corsage 1, you’re ready to move on to thequestions.

1. (C) Here it is, the acceptable selection question that I’m sure you’ve grown to know andlove. Don’t be intimidated by the amount of space it takes up on the page; simply do whatwe do for every acceptability question — take the rules, one at a time, and scan down thelist of choices to eliminate the ones that don’t conform. Let’s start with the easiest rulesfirst, because they’re the easiest ones to check. Any choice with no rose in corsage 2? Yup,(E), and that violates Rule 2, so it’s history. What about choices that mistakenly leave out aG in 3? Scanning the list, I see none of those, but choice (A) has an orchid in 3, which Rule3 forbids, so it’s out too. Now, if you have the correct interpretation of Rule 1 in mind,choice (D) and its three flower types stick out like a sore thumb, so it has to go, leavingonly (B) and (C) as contenders. (B) fails on two counts: One, there’s no violet represented,and two, there are four orchids and three roses, a blatant impossibility. (C), by default, iscorrect, and there’s no need to even check it. If you scanned the choices as we havedescribed, methodically focusing on each rule, one at a time, this question should havetaken little more than seconds.One other thing to point out here: Don’t worry if every aspect of the game hasn’t fully“sunk in” when you move on to the questions. Even the best logic gamesters may feel alittle hazy when they move from the setup stage into the question-answering stage. Thegood news is, working through the first few questions usually helps clarify the rules andaction of the game, and you’ll find that often your knowledge and familiarity of the gamewill grow as you progress through the questions. Don’t get me wrong, always try to do asmuch as you can up front, but be aware that you can use the first few questions to helpyou get up to speed on a game.

Page 38: Prep Test 9

KAPLAN LSAT PREP _______________________________________________________ LSAT Test IX Explained: Section III

36 © K A P L A N

2. (D) There’s one quick and easy way to compute maximums — shoot for the moon. Ifyou try the highest number first, and it works, then that’s the answer and you will havesaved a goodly amount of time. Conversely, when asked to find a minimum, start at thebottom and work your way up until you reach the answer. (This is not necessarily the bestapproach when you're given a hypothetical, as we'll see in Question 3. But it's great whenno additional data are provided.)So, can there be seven roses? No, because with nine slots to fill, that would leave only twoslots and we need to get at least one G, O, and V in there too. What about six roses? Thatwould leave just enough space for one gardenia, one orchid and one violet. Can that bemade to work? Sure — there’s more than twice the number of roses to orchids, everyflower type is accounted for, and we can easily satisfy Rules 1, 2, and 3. So once we reachthe number six, choice (D), and see that it’s possible to have six roses, we have our answer.No need to go any further, just pencil it in and move on.

3. (B) With the addition of a hypothetical, Question 3 repeats Question 2: Find amaximum, this time of violets, given that corsage 1 is set; it gets two orchids and a rose.That reference to “two orchids” has to bring one thought to your mind — a minimum offour roses is required. Whenever a game involves numbers, stop every once and a whileto count things up and see what you've got. So far it's two orchids, at least four roses, andone gardenia (from Rule 3) , for a total of seven flowers so far. Just two slots remain, andthere's nothing stopping us from filling them with violets, so two violets is the max, choice(B).Notice that this time, the best approach was to deduce the answer directly, rather thantrying out the choices as we did in Question 2. That's because this time we were given an if-clause that went a long way towards determining the full distribution of flowers intandem with the rules. By the time we were finished, there was nothing left to chance; weknew the maximum number of violets was two. The point here is, don’t blindly limityourself to a certain approach just because you think that it’s the way the questionshould be done — that’s too formulaic and may get you into trouble. Keep an open mindat all times, and never let rote action supersede the thought process involved in takinghypothetical information as far as you can. If you do that, as in this case, you may verywell stumble on the answer without requiring additional steps.

4. (A) If corsages 2 and 3 are identical, then since 2 has a rose, 3 must have one also. And 3has a gardenia, so 2 must get a G as well. Corsage 3, according to Rule 3, has no orchids, socorsage 2 must be orchidless too. But the game requires the use of at least one orchid, andneither 2 nor 3 can have one, so corsage 1 must have at least one O. So far, then, thesituation looks like this: Corsage 1 has an O and two slots yet to be filled, while corsages 2and 3 each have a rose, a gardenia, and one slot left to fill.After engaging in a fair amount of deduction, it's wise to take stock of the question andsee if it's time to attack the choices. The stem, subtly worded, wants an exact number andtype of flower that can be included. We've deduced so much already, we may as well tryout the choices, and you can think each through, helped by your pencil. (A), for instance:If there were exactly two orchids, both could only be placed in corsage 1, requiring at leastfour roses, or two more than we have already installed. The two additional roses could beput in corsages 2 and 3, which remain identical; and the single required violet can be thesecond flower-type in corsage 1. We see that it is indeed possible to use exactly twoorchids, so (A) is right and we stop there. You simply must develop the confidence in yourwork to take a correct answer as soon as it presents itself. On Test Day you probably

Page 39: Prep Test 9

KAPLAN LSAT PREP _______________________________________________________ LSAT Test IX Explained: Section III

© K A P L A N 37

wouldn't bother to confirm that five violets, choice (E) is out of the question (as there areonly four spaces left to fill); or that a total of five roses (D) forces a violation of one rule oranother, ditto exactly three gardenias or exactly three roses, choices (B) and (C).

5. (C) That two of the corsages have at least one orchid is just coy. Why not just state thatcorsages 1 and 2 each have exactly one orchid? Because, of course, they're testing whetherwe remember that corsage 3 never gets an O, so the two corsages they’re referring to mustbe 1 and 2. And once an orchid is placed in 1 and 2, that’s all for the Os, because our earlieranalysis determined that two Os is the maximum. Amazingly, that’s enough to answer thequestion: Corsage 2 has to contain at least one orchid, as just mentioned, and the rose that’sbeen pinned there since the beginning of the game. Voila — choice (C). As for the others,there’s no pressing need for a gardenia in corsage 2, killing choices (A) and (B), or a violet,for that matter, killing (D) and (E).

6. (D) The maximum issue pops up again, this time as a hypothetical mandate to use asmany violets as possible. Remember, when you’re seeking to maximize the occurrence ofone entity, you must minimize the occurrence of the others. A minimum of one orchidand two roses will satisfy, as will one gardenia. Those total four, leaving five violetsmaximum. Be sure to check whether those numbers can be acceptably placed intocorsages — which they can, easily; there's no danger of violating any rules — so those arethe proper numbers, and the correct answer must be some data out of "one O, two R, oneG, and five V." Only (D), "two R," satisfies.

7. (A) Whenever there's a lot of information around, always start with whatever isconcrete. Here, you can create a sketch for Question 7 and immediately insert the newminimum one G and one V into corsage 1, as well as the usual R in 2 and G in 3. Nextwe're told that three types of flowers must make up corsage 3, and 3 of course takes noorchids, so the rest is evident — G, R, and V for 3. As for orchids, two is out of the question(we cannot accommodate four roses without forcing three flower-types, G V O, intocorsage 1, in violation of Rule 1). So a single orchid will have to be used, and in corsage 2since (again) putting an orchid into corsage 1 would violate Rule 1. Now, considering thatthe question asks for a group of flowers that could make up corsage 2, it would be a safe betto axe any choice that doesn’t contain a single orchid, and happily that includes all of thechoices except correct choice (A). Note, by the way, that "one rose, one orchid, and oneviolet," or "one rose, one orchid, and one gardenia," or "one orchid and two roses" would allbe equally acceptable answers here.

Page 40: Prep Test 9

KAPLAN LSAT PREP _______________________________________________________ LSAT Test IX Explained: Section III

38 © K A P L A N

Questions 8 - 13

As mentioned in this chapter’s introduction, Game 2 is a grouping game that some founddeceptively difficult. That was partly because it looks so standard. Choosing four peoplefrom a simple group of seven is a familiar task, and even the four rules look like a cinch:We have to pick J or K but not both; N or P but not both; N requires L; Q needs K. Thisvery familiarity may have lulled test-takers into a false sense of security. Certainly, thosewho merely jotted down the rules as described above and, without any further thought,went straight for the questions, were in for a rude awakening.It was mentioned earlier that most of these six questions are very vague. The first threegive us no information to go on, and that can really trip you up — it places a heavy burdenon you to make things happen. Granted, Question 8 is an acceptability question, and canbe handled readily with the Kaplan technique discussed in the previous game: Check thechoices against one rule at a time, eliminating the ones that don’t conform. But after that,9 is vague, 10 is vague; 11 and 12 supply hypotheticals, but still require a good amount ofwork if you haven’t done your homework up front; and 13 is a classic “backwardsreasoning” question, a type famous for strenuously testing one's understanding of thegame. So let’s go back to the rules and see what we can do about thinking through themand creating a framework that will help us get a better handle on the game.We warn you in advance that we're about to lay out all of the game's possibilities in a fairlycomprehensive way. We also admit that there are ways of succeeding on this game, andothers like it, other than mapping out the possibilities as thoroughly as we will here. But ifnothing else, this will be a useful exercise in deduction and in taking the rules as far as wecan, and that should be a high priority for any Kaplan student. (If you turn to the last pageof this book, you'll see a sketch that reflects both the end product of this process and theindividual steps leading up to it. If you refer to it, you may find the following discussionclearer.)Rule 1 immediately gives us two possible quartets — one including J, and one including K.Remember, J and K can't both be chosen. So no matter what else happens, one and onlyone of those two, J or K, will be selected. Meanwhile, Rule 2 imposes another requirementon the selection process, the need to include either N or P but not both. So we can begin bysetting up four basic possibilities, exactly one of which must be chosen for the quartet: It'snecessary to choose either J N, K N, J P, or K P. We can call those Options 1, 2, 3, and 4respectively. Yes, you can label the options for yourself just that way, even on Test Day.You could jot them down with dashes representing the slots still open: J N — —, and so on.You have to realize that exactly one of those pairs will be chosen, along with two moredinner guests to be picked out of the group of L, M, and Q.Hang in there, because the remaining rules help us narrow down the possibilities evenfurther. One cannot choose N without choosing L, says Rule 3. Therefore, we can add totwo of our options. Option 1 must consist of J N L —, and 2 would be K N L —.Nexr, Rule 4 says that choosing Q requires choosing K. Yet by definition, K is not part ofOptions 1 and 3 (J is there instead), so Q is forbidden to those options. To remember that,ought you to write “NO Q” next to Options 1 and 3? Well, it wouldn't be incorrect to doso...but it's not the cleverest step you can take. The cleverest step is to remember thatwhenever you learn some information about what is not so, it's to your advantage to givesome thought to what is so. In other words, try to turn negative information around tothe positive; it’s much more powerful in that form. Take Option 1, which now reads J N L— . Ask yourself: Who can take that fourth slot? Not K or P, we know; and not Q.Therefore M must be that fourth person; and the complete option reads J N L M. That's

Page 41: Prep Test 9

KAPLAN LSAT PREP _______________________________________________________ LSAT Test IX Explained: Section III

© K A P L A N 39

one possible quartet. If (following Rules 1 and 2) J and N are selected, then J N L M will bethe full quartet.The same goes for Option 3, currently J P — —: With K absent, Q is ineligible, so theremaining slots would have to go to L and M: J P L M. Two complete, possible quartetshave been assembled.Finishing off Options 2 and 4 isn't quite as tidy, but there's still progress to be made. Whenwe last saw them, Option 2 consisted of K N L —, and Option 4 was K P — —. For each,Person Q is eligible to be chosen for the dinner party, since K is present, but we're notrequired to choose Q. So for Option 2, we can jot down that either M or Q will take theremaining place (remember, since K and N are picked here, J and P are ineligible). Finally,Option 4 is the least firm of all: K and P are chosen, and thus J and N are out of luck,butthe other two slots can go to any two out of L, M, and Q. No way to narrow it down furtherwithout more information.Is this at all confusing? Does it sound like a lot of work? Maybe so; but watch howquickly the questions fall now that we have the four possible options mapped out on thepage as follows:

1) J N L M

2) K N L — [M or Q]

3) J P L M

4) K P — — [two of L, M, Q]But don’t get discouraged if you didn’t think to work all of this out on your own; you’reprobably in the majority. Besides, the point in reviewing a PrepTest is not to beatyourself up about what you got wrong or didn’t see. It’s precisely to learn things that youcan thereafter incorporate into your own approach, so that on your Test Day you'll havethe most potent arsenal of techniques and tips that you can assemble. So, while you neednot go as far as we have gone in mapping out all of the possibilities, it was worthwhile totake you through the thought process. Maybe it'll help some of you accept that, as ageneral rule, the best Logic Gamesters are those who always try to take the rules as faras they can — to learn as much as they can before moving to the questions.

8. (D) As mentioned earlier, this acceptability question shouldn’t be difficult no matterwhat approach you take. Scanning the list, we can quickly kill choice (A) which includesboth J and K contrary to Rule 1, and (E), which includes both N and P in violation of Rule2. N requires L, but doesn’t get it in (C), and Q is illegally chosen without K in choice (B).That leaves (D), which (you may notice) turns out to be our Option 4.

9. (B) The unprepared student might be baffled by this question, but armed with our fouroptions, it becomes a breeze. First, you have to make sure that you've clarified thewording of the question for yourself. Otherwise, time is wasted and accuracy is placed injeopardy. We are looking for a pair of people, one or both of whom has to be included in

Page 42: Prep Test 9

KAPLAN LSAT PREP _______________________________________________________ LSAT Test IX Explained: Section III

40 © K A P L A N

the dinner party. It follows, therefore, that the four wrong choices are pairs of people whocould both be excluded. We need to attack each choice in turn, seeing whether we can find agroup of four that excludes both of the people named in the choice. If there is, then thatchoice can be eliminated. This process is sometimes called " proving the exception."And if you start at the top, you don't search for long. Our Option 1, the group consistingof J, N, L and M, eliminates choice (A) — and (E), for that matter, if you ever got there,since both choices mention pairs of people who don't appear in that possible quartet. Butas you scan the four options, you see that every one of them does in fact include L or M orboth, so (B) is correct. For the record, Option 4 allows for the group K, P, L, Q, whichshows that, contrary to (C), we can leave out both N and M; and Option 3's J P L M leavesout both N and Q, so (D) is wrong as well.

10. (B) Let’s look at two ways to approach this one. First, we can use the principle of"proving the exception" to eliminate the wrong choices, by comparing each choice to ourfour possible options. Here’s how it works, starting with (A). Question: Is J and N the pairthat can't both be selected? Answer: See whether J and N can be selected together. If so,then they’re not the answer we seek. Well, Option 1 clearly indicates that J and N can beselected together, so (A) is wrong. What about J and Q, choice (B)? No, J is only found inOptions 1 and 3, and Q is not selected in either, so we have our answer. We tried to provethe exception, and couldn’t; therefore, that choice must be correct. A quick glancethrough the rest of the choices and the options will show that the people in the remainingchoices can be selected together.Another way to go about this question involves using the contrapositive. As you probablyknow, any if/then statement can be rewritten, with equal validity, by reversing andnegating its terms. This concept, called the contrapositive, is covered in Kaplan's LogicalReasoning lessons, and comes in handy for Logical Reasoning as well as for Logic Games.The contrapositive of Rule 4 is that if K is not selected, then Q is not selected. But if K is notselected, says Rule 1, then J must be. So here’s what we have: If we pick J then we can'tpick Q, and if we pick Q, then we pick K instead of J. It is therefore deducible from thevery beginning that J and Q will never be seen with each other at the dinner, and again (B)has it right. Incidentally, the contrapositive to the third rule is “If no L, then no N,” andyou might notice that we did work with this concept, even without attaching a fancy nameto it, when constructing the possibilities earlier.

11. (B) This one is actually pretty involved: We’re told to exclude M, and determinewhich of the five pairs of people must therefore be included. The question could beparticularly troublesome for those attempting to answer it from scratch, because there areno rules directly involving M: We can't deduce much from the selection of M, whoappears where he or she does based on how the rules affect the other entities. Those whodon’t make an attempt to work out possibilities early on get stuck doing so now; and letme tell you, there’s a lot more pressure on you as you attempt to rethink an entire gameat a late stage, than while you're setting it up on the front end. Lucky for us, we have allthe information we need to answer the question right there on the page.If M is excluded, the possibilities are narrowed down to Options 2 and 4, since the othertwo options definitely include M. In the absence of M, Option 2 would result in the groupK, N, L, Q, while Option 4 would become K, P, L, Q. That’s it. With the help of all of ourwork up front, it takes literally a few seconds to determine everything we need to knowto answer the question. This is what justifies the time we invested earlier. The only pairamong the choices that’s included in both possible quartets is K and L, choice (B).

Page 43: Prep Test 9

KAPLAN LSAT PREP _______________________________________________________ LSAT Test IX Explained: Section III

© K A P L A N 41

12. (C) If P is excluded, then we’re dealing with Options 1 and 2 only, the ones thatinclude N instead of P. Option 1 is fully determined to be J, N, L and M — that's onepossible quartet; while Option 2 consists of K, N, L, and one space left open for M or Q.That constitutes two more possible quartets — K, N, L, M, and K, N, L, Q — for a total ofthree.

13. (E) The "backwards-reasoning" question gives us a desired consequence, and thenasks us to determine the circumstances that would bring it about. In essence, it's askingfor a new rule that, combined with the other rules, will mandate a particular result. Onceagain, the principle of proving the exception can help us knock off the wrong choices.Since the desired result is one and only one acceptable group, and the right answer is apair of people whose selection will bring that about, it follows that selecting any of theother four pairs will result in the possibility of more than one acceptable group.So with each choice, try to prove the opposite. If choice (A)'s J and L, for instance, can beincluded in more than one acceptable quartet, then (A) doesn't have the desired effect.And scanning our trusty options, we see that both Options 1 and 3 include that pair. So(A) is wrong: Choosing J and L doesn't limit the group to one possible foursome, so (A) iswrong. K and M, choice (B), or L and Q, choice (D), could be part of different quartetsusing Options 2 and 4. And L and N are included in different groups in options 1 and 2.But if M and Q are chosen, then the only acceptable foursome is K P M Q — from Option 4.So (E) is correct.

Page 44: Prep Test 9

KAPLAN LSAT PREP _______________________________________________________ LSAT Test IX Explained: Section III

42 © K A P L A N

Questions 14 - 18

Three boys...three girls...three dances...It’s time for “Dance Recital”! Game 3 is a classicgame of matching: Before the brief stimulus paragraph is over, it should be clear to youthat your task in this game is to match up the dance partners and the dance numbers. Andif it wasn't clear, then skimming the questions and answer choices certainly should haverevealed that that was the case.Once you've determined the game's task, another key component of the Overview step isto give some thought to the numbers that govern the game. In terms of this particularsituation, with three boys and three girls dancing, it probably should've crossed yourmind to ask: Does each kid have to dance with each of the others, or can partners staypaired for two dances? That's why Rule 3 is probably the most significant for us — it tellsus clearly that yes, each boy dances once, and only once, with each girl...and vice versa. Thisbecomes enormously useful in the course of each question, as we use a process ofelimination to determine who must partner whom.You simply have to know this rule, Rule 3, cold. If you fumbled in the course of the game,it was probably because you fumbled the implications of this rule. Stay tuned.First, however, how to keep track of the data? In matching games, there are fewscratchwork tools that are more useful than creating simple lists, and this game is noexception. You can set up a column for each of the three dances, and list the pairs as youdiscover them, using CAPS for one gender and lower-case for the other. In fact, if you turnto the last page of this book, you'll see what we mean. Better still, by flat-out listing thethree girls in each column, you get to build Rule 2 right into the picture. This is useful. Infact, whenever you can build a rule right into the picture, you should do so: It makes therule more vivid and close at hand. Throughout the game a sketch like the one at the end ofthis chapter will remind you that Rita's partner in #2 is Sarah's partner in #3, a rule thatwould be tough to keep in mind without some sort of visual aid. (Of course you could listthe boys in each column and match the girls to them, except that then it's harder to buildRule 2 into the sketch.) Notice that Rule 1 is easily indicated as well.In the end, however, success on this game has less to do with the nature of the sketch thanwith your ability to deduce what must be true in each question by process of elimination.Students often have trouble with this, so pay close attention to the thought process wedescribe for each of these five questions.

14. (D) Keeping an eye on your master sketch, you might quickly re-draw the fourcolumns, inserting a lower-case l for Luis next to Sarah in dance #3. Your visual cue toRule 2 pays off, instantly reminding you that Luis, as Sarah's #3 partner, will partner Rita in#2.Here's where you have to step back and look at Luis. Paired with Sarah in #3 and Rita in#2, he simply must dance with Tura in #1. (See what we mean about knowing Rule 3 cold?Each boy must dance with each girl once. Fumble that, and your work is paralyzed.)Anyhow, with Luis dancing with Tura in #1, Rita and Sarah will take a turn in #1 with Karland Miguel, though we can deduce nothing more about the matchups. Sarah can fulfillRule 1 by dancing with Karl in either #1 or #2, so both girls are available to dance withMiguel in #1, as (D) has it.

Page 45: Prep Test 9

KAPLAN LSAT PREP _______________________________________________________ LSAT Test IX Explained: Section III

© K A P L A N 43

15. (B) If you draw another set of columns here and insert Miguel as Rita's partner in #2,again your visual cue for Rule 3 should tell you instantly to match Miguel with Sarah in#3. And don't begrudge the creation of another little sketch — it really takes no time atall and, as long as you're recopying your master sketch, no real brainpower, either. Thistime, step back from Miguel: His dances with Rita and Sarah accounted for, he has todance with Tura sometime, and that sometime will be dance #1.Nothing else come to mind? When no other deductions are apparent, it's probably timeto check your answer choices. And here, four of the five choices blatantly contradict whatwe've deduced about Miguel. Only (B) doesn't contradict our data — there's no barrier toLuis dancing with Sarah in #2, so (B) is that which "could be true."

16. (B) The focus of this question is dance #3, so keep an eye on it. And notice that, for thefirst time, a question allows us to apply the simple first rule directly.With Sarah forced by the question stem to dance with Miguel in #1, she's forced by Rule 1to partner Karl in #2 and thus, by process of elimination, to dance with Luis in #3. Rule 2says (as our master sketch should remind us) that Sarah's #3 partner is Rita's #2 partner, sowe assign Luis to Rita in #2, leaving Tura to dance with Miguel in #2. Miguel hasn'tdanced with Rita yet, so he will do so in #3, leaving Tura to dance with Karl in #3. That'sthe pairing that the testmakers happened to put into (B) as the correct answer.

17. (C) But for the fact that the focus is now on dance #1 instead of #3, this one is similar toQuestion 16. It also begins the same way, with the application of Rule 1. Assigning Luis toSarah in #2 (as the stem does) mandates that Karl partner Sarah in #1, leaving Sarah todance with Miguel in #3. Rule 2 kicks in, of course, allowing us to note that Rita will dancewith Miguel in #2.By this time, we surely ought to realize that whenever two of a kid's partners areconfirmed, we can confirm the third. Since Miguel is now assigned to Rita in #2 and Sarahin #3, clearly his turn with Tura comes at dance #1...leaving Luis as Rita's partner in dance#1, choice (C).

18. (D) This one is considerably tougher, since the stem doesn't have the same immediateimpact as the others did. Assigning Miguel to Rita in #1 doesn't seem to mandate any otherparticular pairing right away. Well, when trying to decide how to proceed, consider therule or rules that seem most influential. Here, Rule 2 has the edge. With Rita'sresponsibility to Miguel out of the way, it'll be either Karl or Luis who partners Rita in #2(and, therefore, Sarah in #3). So try it out both ways, and if you do, you should realize thatthe former is impossible: If Karl were to dance with Rita in #2, he'd have his one and onlydance with Sarah in #3, but that would violate Rule 1. Nope, it's inevitable that it will beLuis who dances with Rita in #2 and Sarah in #3, reflected in correct choice (D). If youwere to work out all of the other pairings (though who wants to?), you'd see that (A), (B),and (C) get Karl's partners all wrong — it's Sarah in #1, Tura in #2, and Rita in #3; whileLuis partners Tura in #1, not #3 as (E) would have it.

Page 46: Prep Test 9

KAPLAN LSAT PREP _______________________________________________________ LSAT Test IX Explained: Section III

44 © K A P L A N

Questions 19 - 24

Notwithstanding the fairly elaborate sketch, which suggests a spatial arrangement game,“Hospitals, Jails, and Universities” turns out to be pretty much a straightforward groupinggame of distribution — one in which entities are placed in subgroups.Still, games in which the testmakers provide any sort of picture tend to put off somestudents, because the sketch provided is usually somewhat out of the ordinary and thus abit scary (cf. the parking lot game in PrepTest VIII for another example). But what youneed to know is that when the testmakers provide a sketch, generally the game becomeseasier. They give you a sketch mainly because it would be too difficult to describe thesituation in prose! In terms of this game, consider the rhetorical hoops they'd have to gothrough to describe in words those little squares at the inside corners (whose sole purpose,as we'll see, is to separate Cities 2 and 3 for the purpose of making a "big deduction"). Bypresenting facts about the game in picture form, the testmakers do us a favor.I hope you didn't waste time studying the details of the drawing, however, but ratherfollowed our perennial advice to start by defining the game's real-life situation and yourtask within it. Doing so quickly reveals that these six cities have a total of eightinstitutions, and your job is to place the eight entities where they belong. Again, thisplacing of entities in subgroups is the essence of grouping games of distribution, and are afamiliar task. (To be sure, the game does turn out to have a single, but important, spatialelement; see our discussion of Rule 5, coming up.)Certainly, when the testmakers are thoughtful enough to provide a sketch, it's only politeto take advantage of it. Build your "master sketch" right there at the top of the page.And of course, set up nearby your roster of entities — H H H H J J U U — so that youcan knock them off one by one. Before going further you should establish the parametersof the task. For instance: Does every city have to receive an institution? Skim the rulesand you'll see that that's not so — there's no requirement that every city have a letterassigned to it (not until Question 22, anyhow). Also, you might notice that Rule 1 acts as aloophole closer, confirming (if the idea struck you) that none of the institutions is sitting onthe border between cities. Always be alert for "loophole closers," and remember thattheir purpose is to limit possibilities that might otherwise be alive. They don't usuallyhave to be jotted down, but they have to be heeded.What about the job of assigning letters to cities? Remember that you need not attack therules in the given order. Start with the most concrete ones. It's fine to look over all therules quickly; but when the time comes to work for real, you'll get off to the best start ifyou locate the most concrete information. And what could be more concrete thanindented Rule 6: You are to place a U in City 3 and a J in City 6. Do so; and then see whichother rules tie in with that one. (We're really encouraging you to attack the game toachieve a goal — don't just "read" it passively!) Your eye probably lit on Rule 4: If as itsays, you can't have a jail without a hospital, clearly one of the H's must be placed in City 6along with its J. Insert that — and perhaps note, off to the side, that the remaining J will alsohave to be paired with an H when the time comes.Rule 5 should also catch your eye, since there are only two universities and one of them isplaced (in City 3). Given that the cities containing U's cannot share a common boundary,you must ask yourself: Where can the other U go? As we've said so very often: Whenevera rule tells you what is not true — like Rule 5, here — it's up to you to draw the positiveconclusion: What must or could be true? The only cities with which City 3 does notshare a common boundary are Cities 2 and 6. Can City 3 itself take a second university?Check the rules; you'll realize that Rule 2 forbids that. Check further, and realize that City

Page 47: Prep Test 9

KAPLAN LSAT PREP _______________________________________________________ LSAT Test IX Explained: Section III

© K A P L A N 45

6, already possessing a jail, is barred by Rule 3 from having a university. The upshot is the"Big Deduction" for this game: The other U must be assigned to City 2, and you shouldindicate that in your sketch.From this point on, the game becomes a straightforward grouping game of distribution, sinceyou're simply putting entities into subgroups. (Never again, as it happens, do we have toconsider the spatial element.) And four of the eight entities have been assigned, so off tothe side you should have noted what's left: H, H, and the JH pair required by Rule 4.There's nothing left to do but go on to the questions, but don't draw another sketch unlessyou need to: Chances are you can pull some points out of the work you've done at theoutset.

19. (E) Having made the "Big Deduction" described above, you might have figured thatyou'd be asked right away "Which one of the following must be true?" and the answerwould be: "City 2 contains a university." Surprisingly that doesn't happen, but we do geta couple of gimmes anyway that require you to have made that realization. Checking themaster sketch against the choices for "what could be true," we see that neither (A) nor (B) ispossible, because the two U's are definitively assigned to Cities 2 and 3; and since thosecities contain universities they are forbidden to house jails, so (C) and (D) are false. Thatleaves (E), and indeed City 3 could take a hospital, though it need not. And while we're onthe subject....

20. (A) ...City 1 could take a hospital, too, so the first answer choice is the correct one forQuestion 20. Note that not only are Questions 19 and 20 worded identically, but they sharethree wrong answers: 19(A),(B), and (C) are pretty much the same as 20(D),(E), and (C),respectively. How about that! As for 20(B), of course City 1 contains exactly zerouniversities.

21. (D) When we last looked at the roster, remaining were a J and three H's — andremember that an H has to accompany that J. Question 21 wants the full list of possibilitiesfor placing that J (and it's often a good idea to scan the choices. When you do, you seethat City 4 appears in all five choices, so you needn't worry about City 4). Next, realize thatsince City 2 is assigned a university, jails are forbidden to that city by Rule 3, so eliminatechoices (B) and (E), each of which include City 2. Any problem with locating theremaining J in City 1 or City 5? None whatsoever. So (D) is the complete list.

22. (D) Here, for the first time, you might have wanted to draw a simplified sketch for thepurpose of a single question, inserting a dash in Cities 1, 4, and 5 to indicate that each mustreceive at least one letter. (Question 22's minimum requirement, of course, is alreadysatisfied by Cities 2, 3, and 6.) Available to put into those dashes are three H's and a J —and remember that that J is never alone, but teamed with an H. It follows that all three ofthose cities, 1, 4, and 5, will receive at least one H, and that makes (D) the statement that"must be true." Of the wrong choices, (A) and (E) are only possibly true (in fact, if City 5gets the jail, then both are false), whereas (B) and (C) are false on their face: Assigning ahospital to either City 2 or City 3 would make it impossible to meet the condition set byQuestion 22's stem.

Page 48: Prep Test 9

KAPLAN LSAT PREP _______________________________________________________ LSAT Test IX Explained: Section III

46 © K A P L A N

23. (B) It's useful to recall that every game has low, medium, and high difficulty questions,and when the game is straightforward, they make the questions difficult by engaging intricky wording. In its shy way, Question 23 is telling us that four of the five choices' citiescould contain three or four hospitals — those are the wrong answers; while the correctchoice names the city that is doomed to zero, one, or two hospitals. What limits theplacement of hospitals? Rules 2, 3, and 4, which tell us to pair each J with an H, but neverto put two J's together or a J and U together. If there are three or more hospitals in a citythat contains a university, figure it out: We wouldn't have enough H's to go around. Cities2 and 3, of course, are the ones with universities, so either could star in the correct answer;they chose the former for (B). The other cities named in choices could, in fact, contain threeH's, so long as a J was also present.

24. (A) Another one with tricky wording. As always, begin with what's concrete: Thestem says that a city contains "H H U," and that must of course be City 2 or City 3, the onlycities with U's in the first place. We're asked for a list of three cities that could, under thosecircumstances, contain no hospitals. I worked out the two concrete possibilities on paper,but (as luck would have it) stumbled across the right answer before going all the waythrough it.If City 2 is the one containing "H H U" (which it could be), then we'd have a U in City 3 andJH in City 6, leaving only the remaining "JH" pair, which would have to go into City 1, 4, or5. Under those circumstances, the cities with no hospitals would be either 1, 3, and 4; 1, 3,and 5; or 3, 4, and 5, depending on where "JH" went. And as it happens the second of thosetrios is mentioned in choice (A), which must be the correct answer! You don't even have towork out the other possibility, namely the one in which City 3 receives the "H H U." There,the trio without hospitals would be either 1, 2 and 4; 1, 2, and 5; or 2, 4, and 5. The point isthat any of the six trios in italics in this paragraph could have been chosen as the correctanswer to Question 24, and the sooner you locate it, the sooner you're done.

Page 49: Prep Test 9

© K A P L A N 47

SECTION IV:

LOGICAL REASONING

Page 50: Prep Test 9

KAPLAN LSAT PREP _______________________________________________________ LSAT Test IX Explained: Section IV

48 © K A P L A N

1. (C) The flaw should have leaped right out at you — the author is comparing apples andoranges here. People shouldn’t complain about the supposed incompetence andinefficiency of the post office, he says, since a letter from a friend is such a valuable object.Huh? What has the joy of receiving a letter to do with crummy service? The author wantsus to accept without question a rate increase because getting a letter is worth the money,but as (C) says, the author is confusing the value of the process of delivery with the valueof the actual letter being delivered, and this is the egregious mistake.(A) wants us to think that the author believes the postal service is competent and efficient,but this just isn’t so. He merely suggests that we should overlook such faults, since theobject received is so valuable — he never asserts that the service is exemplary. As for (B),the level of the increase is immaterial here, since the flaw resides in the confusion of theservice with the object in question. We don’t need to know what the author wouldconsider an outrageous price, since the price isn’t the crux of the argument. (D) brings upan appeal to an outside authority, but the author never claims that an authoritative voicesupports his assertion. And (E) is similarly bizarre since any connection of the critics to thepostal service is immaterial — it doesn’t have any relevance to the flaw.

2. (E) Medical journals again — remember Question 25 from Section II? We need toweaken this argument, and here we’ll take the tactic of undermining the conclusion byproviding alternative evidence. The author claims that if results concerning a study onheart attacks had been published sooner, then heart attacks that occurred during the delaymight have been prevented. We can weaken this conclusion by finding evidence thatshows that taking aspirin could not have made a difference during the period in question.Choice (E) does this by explaining that it takes two years for aspirin to have an effect on apatient’s risk of heart attack. Thus, heart attacks that took place during this period couldnot have been prevented by the use of aspirin during the delay period.Choice (A) has no effect on the conclusion — no matter how hard the staff worked, theassertion that the heart attacks in question could have been prevented is unaffected. (B)’slab animals, meanwhile, are outside the scope of the argument, so the questionable effect ofaspirin on heart attacks in such animals does not affect the logic. Stomach ulcers, choice(C), are also outside the scope, since they wouldn’t necessarily have an effect on thosepatients who had heart attacks during the delay. (D) is only relevant as an explanation ofthe reasons for the delay.

3. (C) We need to resolve the discrepancy , and this is one time when prephrasing ananswer — trying to think of a possible explanation before looking at the choices —would have served you well. Contrary to expectations, offers of across-the-board airlinediscounts have actually cut profits, even though lots of tickets have been sold. If you thinkabout it, probably people who were going to fly anyway took advantage of the low prices,and thus the airline lost money on them; meanwhile, if the new fares didn’t bring in peoplewho wouldn’t have flown anyway, then the airline wouldn’t make up the losses anotherway. Any kind of speculation of this kind probably makes (C) stand out; it explains thispossibility and resolves the discrepancy.That fewer than 10% of air travelers don’t seek out cheap fares (A) has no impact on thelogic; evidently even that small percentage is enough to sell plenty of tickets, but thatdoesn’t speak to the issue of why profits are down. (B) explains that certain trips havehigher fares, but we’re given no reason to figure in those relative costs, so the whole issue isoutside the scope. (D) is tempting until you realize that it’s not first-time flyers that are theissue, but rather people who wouldn’t have flown at all but for the discounted fares. And

Page 51: Prep Test 9

KAPLAN LSAT PREP _______________________________________________________ LSAT Test IX Explained: Section IV

© K A P L A N 49

choice (E) brings up a problem with creating advertising that explains restrictions whilepromoting the discounted fares. Clearly this wasn’t a problem, since lots of tickets were infact sold. Only (C) solves the mystery.

4. (C) More formal logic with Question 4. Are you comfortable with the way thisstatement is phrased? Feel free to change it around, as long as you don’t accidentallychange its meaning. How about: “A democracy will function well only if the electorate ismoral and intelligent.” Or remember that “only if” signals a necessary condition, so it isthe “then” clause in a simple if/then statement, so it must also be true that “If a democracyis functioning well, then the electorate is moral and intelligent.” And note that inclusive“and” — it’s not an “or.” Think through the contrapositive properly. It must be true that ifthe voters are neither moral nor intelligent, then the democracy won’t be functioning well.Reverse and negate both terms, remember. So (C) has it right.(A), on the other hand, claims that if the electorate is moral and intelligent, a democracywill function well . But the intelligence and morality of the electorate are necessaryconditions, not sufficient ones. It could be true that, although the electorate is moral andintelligent, insane dictators keep trying to take over, and thus a given democracy is havingtrouble functioning well. All kinds of other factors could come in, so (A) is out. (B) iswrong for much the same reason. There could be reasons other than the lack of morality orintelligence on the part of the electorate that could preclude a democracy fromfunctioning well. Do (D) and (E) sound familiar? They reflect the same problem as (A)and (B). All of these choices confuse necessary conditions with sufficient conditions. Youcan turn formal logic inference questions into “gimmes” as soon as you truly understandthe implications of the stimulus.

5. (E) Here, as with Question 1, if you know what you’re looking for in advance, a flawshould become readily apparent, if not readily identifiable, as you read the stimulus.Read the question stem first. First the author compares two groups: infants with normalhearing who can distinguish sounds from any language and not just from the language ofthe people who are raising them; and young adults, who can only readily distinguishsounds from the languages they use. There’s one segment of evidence. The author thenpoints out that hearing begins to deteriorate after infancy. Well, you might say, perhapsthat has something to do with the fact that young adults can’t distinguish sounds as well asinfants can. Perhaps not, though that certainly sounds plausible. Yet the author goes muchfurther, drawing his conclusion based only on these facts,: He asserts that that thedifference between infants’ and young adults’ sound discrimination abilities must be due tothe deterioration of hearing. That’s a big leap in logic, and, as choice (E) explains, theauthor takes a factor that might contribute to an argument, and uses it as a sufficientexplanation.(A) accuses the author of using an arbitrary cutoff point, though we have no reason tobelieve that six month figure isn’t scientifically documented. (B) wants to know moreabout the methods used in the studies, but we can still find the argument plausiblenevertheless. To put it another way, that the author fails to explain the methodology makesit no more and no less likely that those methods were flawed. Clearly we’re not concernedwith choice (C)’s sounds that occur in most languages, only with sounds that are unique indifferent languages. And (D) suggests that the author makes an assumption based on thecharacteristics of a group and applies the assumption to individuals within the group, andthis just doesn’t happen. In fact, the argument makes no assumptions about individuals atall.

Page 52: Prep Test 9

KAPLAN LSAT PREP _______________________________________________________ LSAT Test IX Explained: Section IV

50 © K A P L A N

6. (C) The author is trying to convince us that ending mandatory retirement at 65 wouldavert labor shortages, and implies that the current policy should be lifted in countries withlabor shortages and many productive older workers. Let them work past 65, she says, andthere go your shortages. Of course, she is assuming that the older workers in question willwant to continue to work after 65. The denial test described several times in Section II canbe used here. Remember, if a statement is truly an assumption of the argument, thendenying it must contradict that argument. Suppose that many workers would notcontinue working beyond 65 even if they were allowed to do so? Then the author’s planwouldn’t work, would it? Shortages would be no less prevalent. So (C) is a keyassumption.The denial test helps you deal with wrong answers, too. If (A) is false, if older workers infact have nothing exceptional about them, the argument is undamaged because theproblem is a lack of a work force, not a lack of skilled workers. In the same vein, if (B) isfalse and workers are in factwell prepared “to face the economic consequences of enforcedidleness” (what a phrase!), changing the out-at-65 policy still could solve the labor shortageproblem. Who’s to say that, ready as they might be to cut back their life-style in the face ofmandatory retirement, those folks wouldn’t just as soon working and earning? (D)’scapsule history of mandatory retirement is way off the subject (why the policy was firstcreated has no necessary effect on its wisdom today), while (E), if true, weakens theargument: If in fact there are few unemployed retirees, then removing the age-65 limitwould have little or no effect on the labor shortage problem.

7. (D) Here’s a cheery question about the cancer rates of Japanese immigrants to NorthAmerica. We’re first told that there’s a lower incidence of most types of cancer in Japanthan in North America, even though Japan enjoys a “modern life-style” and industrialpollution. But when Japanese immigrate to North America and start eating like NorthAmericans, their cancer rates go up, until they’re about as high as North Americans’ cancerrates. The best inference is (D), which says that genetic factors aren’t responsible for thelower cancer rates among Japanese living in Japan. This makes sense: If the Japanese wereprotected from cancer by their genes, why would their cancer rates increase after theyimmigrated to North America? After all, it’s not like they left their genes behind in Japan.High stress levels are never mentioned at all, so (B) can’t be inferred. Meanwhile, andtypical of wrong answer choices on inference questions, (A), (C), and (E) go beyond thescope. We know that Japan and North America both have industrial pollution, but do weknow that the level in North America is higher? No sirree, so (A)’s not inferable; for all weknow, there may be less industrial pollution in North America. Although the argumentdoes imply that diet is the key factor, we can’t go out on a limb and infer that the Japanesediet contains actual cancer-curing elements, as (C) claims. Maybe it just contains fewercancer-causing elements than the North American diet does. Nor can we infer that fats inthe North American diet are the culprit, as (E) alleges. Maybe it’s chemicals in the food, orhigher protein levels, or too many Doritos. Again, the argument suggests that diet is theissue, but we don’t know enough to infer anything as specific as (C) or (E). Watch out forwrong choices like these on the test: Stick to what can be inferred based on theinformation given.

8. (C) Don’t let unusually worded question stems throw you off; take a moment to makesense of them. They are asking about familiar Logical Reasoning concepts. Here thereare several ways to interpret the question stem: It’s asking for details; it’s asking for thingsoutside the scope and therefore irrelevant; it’s asking for something that neither strengthens

Page 53: Prep Test 9

KAPLAN LSAT PREP _______________________________________________________ LSAT Test IX Explained: Section IV

© K A P L A N 51

nor weakens the argument — all the kind of stuff you normally don’t want in a creditedanswer! Oh well. This long argument boils down to the statement that when speed is of theessence, a computer is a better translator than a human, because the computer’s work isfaster and more stylistically consistent. The author seems to find an 80% accuracy rateacceptable. 100% accuracy is not essential, and stylistic consistency is, when speed is anissue.Since the author considers stylistic consistency essential, (A) certainly is relevant toevaluating this argument: her preference for computer translation is based on the inabilityof human translators, as a group and working quickly, to produce a consistent document.If human translators could work together consistently, would she still recommendcomputers? The point of (B) seems to be that accuracy cannot necessarily be quantified; ifit can’t be, then “percent accuracy” may not be a useful way of evaluating the programs,and the argument would be weakened. (C), however, steps into irrelevancy. The author isnot recommending using multiple computer programs to translate one document, so thestylistic variations don’t matter. One program would make one stylistically consistenttranslation. On Test Day, of course, your odds of racking up solid points are imcreased ifyou can take the time to check all five choices. Doing so, we confirm that (D) is veryrelevant. After all, if the computer translation renders an drastically inaccurate document,what good is its stylistic consistency? And since accuracy is important to this author, (E)would certainly be relevant: For example, if the computer translations are more accuratethan human translations, the author’s case for computers is strengthened.

9. (B) It’s rare in an LSAT dialogue question that the first speaker gets the chance to replyto the second speaker, but don’t let this throw you — the argument isn’t very complex.Don’t let the numbers and statistics throw you either, since LSAT Logical Reasoning isnot about elaborate calculation. Often, as in this stimulus, the stats have no bearing onthe actual argument. When numbers are the main focus of a stimulus, it’s usually becausethere’s a simple confusion involving rates or percents.Myrna begins by explaining that fat calories should be just 30 percent of peoples’ totalcalories, as opposed to the 37 percent fat calorie average in this country. Roland sez that, ifeveryone did this, a very small number of people, 0.2 percent, would have longer lives,and the average life extension would only be three months. Where do these numbers comefrom? We don’t know. Are they connected to Myrna’s 30 and 37 percent statistics? No, notmathematically; but do we need to calculate anything? No. So don’t worry about it.Roland goes on to explain that maintaining a low-fat diet is a great sacrifice, that the miseryit involves is too great a price to pay for a three-month warranty extension on his life. Inresponse (and here comes the basis for Question 9) Myrna asserts that for every one personwho dies from the consequences of a high-fat diet, a lot more people get serious chronicdiseases from the same consequences. Let’s stay on that thought: She’s telling Roland thatearly death isn’t the only negative side effect of a high-fat diet, effectively accusing him ofnot having considered all the facts. That’s what correct choice (B) explains. Myrna doesn’tchallenge Roland’s facts or claims (A) — she provides additional evidence to make herpoint. She doesn’t challenge his statistics or the source of his information, (C) and (D),either — in fact, she doesn’t even mention them. And as for (E), Roland doesn’t assume hisown conclusion and Myrna doesn’t accuse him of doing so. Incidentally, (E) is actually anaccusation of circular reasoning (as is Question 14, choice (B), see below); but reasoningthat is circular essentially proposes evidence and conclusion that are functionallyidentical. Whatever the problems with Roland’s argument, he does provide independentevidence for his conclusion. Speaking of which....

Page 54: Prep Test 9

KAPLAN LSAT PREP _______________________________________________________ LSAT Test IX Explained: Section IV

52 © K A P L A N

10. (B) We can pretty much ignore Myrna, at least for the moment. Roland’s main assertionis that eating a low-fat diet is too unappealing to be worthwhile, even if it might mean achance for a longer life. This makes (B) a key assumption. Of course, to be on the safeside, you should try the denial test. Suppose (B) is false; suppose a low-fat diet can bemade appealing and satisfying to someone who follows it regularly. Then Roland’sargument falls apart: Why not eat a low-fat diet if it’s that good? Since (B), when denied,contradicts the text, then (B), as is, is a part of the text.Choice (A) as is contradicts of Roland’s argument, since he claims that it’s not worth eatingunpleasant food to live longer, while (A) says just the opposite. (C) goes too far, in that wehave no reason to believe that Roland thinks that a healthy diet is the only potential life-extending element there is. (D), like (A) seems to contradict what Roland says: Hebelieves that a low-fat diet is unappealing, which suggests that he would find a distinct tastedifference between foods with the different fat levels. In any event, his argument doesn’thinge on (D) at all. And as for choice (E), Roland needn’t assume that not everyone eats theaverage diet since his argument rests on a speculation — if everyone ate a low-fat diet, thencertain things would occur. The average diet is beyond the scope.

11. (D) The difficulty here is that “the principle appealed to” isn’t stated; it’s implicit and,in fact, practically undetectable. You could stare at the stimulus for a long time withoutcoming any closer to this particular principle, and so sometimes, as here, the only thing todo is to go to the answer choices and look for something that rings a bell.First, however, be sure that you have the argument down pat. In fact, in principlequestions that involve two conflicting arguments, it’s crucial that you understand boththe argument whose principle you’re asked to abstract, and the opposing argument. Thecritics claim that the current practice of housing great works of art in huge city museums isunfair; the reason they give is that the people who go to these museums already have a lotof important artwork available to them. The counterargument objects to this criticismmainly because it would be impractical to try to distribute the few great works of art morewidely, and also because masterpieces should be seen “in context.” Now it’s hard to detecta principle that covers both objections, but one thing you should grasp — the argumentcountering the critics is intended to show that they are wrong to characterize the currentsystem as unfair. When you look at the answer choices you’ll see that only one addressesthis primary concern: (D), which covers the case nicely. The counterargument says thatthe critics are wrong to condemn the current arrangement as unfair because the alternativewould be impractical and would not be as effective a way of showing the artworks. That’sessentially what (D) says — it’s only reasonable to criticize something as unfair if there’s abetter alternative.Both (A) and (B) speak of providing benefits for as many people as possible — (B) is moreextreme, speaking of the “greatest possible benefit” for the “greatest possible” number.The counterargument simply doesn’t depend on such ideas — indeed both choices expressprinciples we might expect to find the critics appealing to, since their basic complaint isthat under the current system some people are missing out. (C) involves a “forcedredistribution,” which is immediately off the mark because no one is talking about“forcing” anyone to do anything. (E) is a trap, figuring that some of you will read“providing a social and historical context” to mean “showing something exactly as it wasoriginally shown.” That’s not a reasonable reading, and besides, it ignores the main pointof the argument, the question of practicality. Note that only (D) mentions the notion ofunfairness: When you’re having trouble with principle questions, look for the choices thatexpress key concepts and contain key terms that the other choices leave out.

Page 55: Prep Test 9

KAPLAN LSAT PREP _______________________________________________________ LSAT Test IX Explained: Section IV

© K A P L A N 53

12. (C) There are two kinds of accountants involved — those who use adding machinesand those who use computers. And someone using the latter can perform morecalculations in less time than can someone using the former. That’s the evidence. Theauthor then concludes that, given identical usage costs, an accountant with a computer canearn more money than one with an adding machine. Sometimes, with a little thought, youcan figure out what the author must be assuming before looking at the choices. Withpractice, this can help your timing on Logical Reasoning. You might notice that the issueof “earn[ing] more per hour” is brought in out of nowhere — up to then, the author hasbeen dealing with how jobs are done and not at all with how well they pay. But since of thetwo accountants described, the one who does the most calculations is said to earn more, theauthor must assume that accountants are paid by the calculation, and that’s exactly what(C) says.Since the pay issue appears so abruptly, you had to figure that the assumption mustconnect it to the argument proper, so it should’ve been easy to reject (A), which ignores $$$altogether. The other bad choices do bring up $$$ but unacceptably. In one sense, (B) goesagainst the thrust of the argument (the author believes that computer-using accountants getmore done in less time, and yet make more money); in another sense, (B) is just irrelevant,since the author is not concerned with accountants’ diligence. (D), too, tends to counter thelogic — one would expect that the accountant who charged more would be the accountantwho makes more. And the topic of choice (E) is accountants in general, but the authorneedn’t be assuming anything about this larger group.

13. (A) Here's a situation many of us are familiar with, figuring out the company vacationpolicy and how it affects us. When an argument involves both an abstract rule and aconcrete situation — as this one does — you're usually better off starting with that whichis concrete. As in Logic Games, the concrete is easier to deal with. Jennifer is going totake four vacation weeks this year, and is entitled to them. However, according to thesecond sentence, for her length of service (three years plus) she gets only three weeks. Aparadox. What gives? Well, she must be taking advantage of the provision that 1/2 of lastyear's unused vacation can be carried over. Meaning that the fourth week that she plans totake this year must be 1/2 of last year's unused allotment, and that makes (A) deducible.Two of the wrong answers are awfully slick. With regard to (B), next year Jennifer willhave worked at KVZ past the four year mark, at which point she may be entitled to morethan her current vacation allotment. And even if she remains at three weeks, she couldchange her stated plans and take a shorter vacation this year, in which case 1/2 of whatremains could be carried over, giving her more than three weeks next year. As for (D),remember that an employee may carry over up to 1/2 of last year's unused vacation. ThatJennifer is asking for one carryover week (which is of course 1/2 of two weeks) doesn'tmean that she actually took the third week as vacation. For all we know, Jennifer never gotaway at all last year, in which case she'd be allowed to apply 1-1/2 weeks to this year but isnow choosing to take only a week.(C) and (E) are classic scope errors. The paragraph is about Jennifer's vacation relative to ageneral vacation policy; nothing about other employees' vacations (C), or the motivationbehind a policy (E), is discussed or can be inferred.

14. (A) The author tells us that the most important safety improvements in the last 20 yearsin the realm of anesthesia-related deaths resulted from better anesthetist training. Makessense; so far, so good. We then learn that, during the 20 year review period, certainmonitoring equipment wasn’t available in most operating rooms. Therefore, the author

Page 56: Prep Test 9

KAPLAN LSAT PREP _______________________________________________________ LSAT Test IX Explained: Section IV

54 © K A P L A N

concludes, the increased use of this equipment won’t cause a significant drop inanesthesia-related fatalities. Wait a minute — that doesn’t make sense. That’d be like saying“Putting salt in the soup made it taste better last time. I didn’t put pepper in last time.Therefore, putting pepper in this time won’t make the soup taste better.” (That wouldn’tbe a bad “parallel logic” answer choice, come to think of it.) Just because one factor led toa result doesn’t mean that a second factor couldn’t lead to the same result — as (A) pointsout.This argument is bogus but not circular; for (B) to be the answer, the author would have tobe saying that “the equipment won’t save lives because the equipment won’t save lives” (orwords to that effect). However, the way it goes now, the author is saying “the equipmentwon’t save lives because it wasn’t present when lives were saved” — a different thing entirely.(C) alleges that the author’s claim is that the improvement in safety was a result of the lackof monitoring equipment, but she doesn’t imply this at all. There’s no inconsistentinformation here, choice (D), since the two pieces of evidence don’t conflict — it’s just thatthe author sees the first as exclusive of the second. And choice (E) is just confusing. Yes,the author does seem to claim that the first event, the better training, caused the conclusion,the drop in fatalities, but there’s no reason to believe that both events were caused by a thirdevent.

15. (D) As we’ve seen on several occasions in Sections II and IV, an extremely effectiveparallel logic technique is to locate and identify a key part of the argument (usually theconclusion), and then see which answer choices do and do not match up. Here it's easy tolocate the stimulus's conclusion thanks to the signal “thus” — always be on the lookout forstructural signals! — and it goes like so: “Engaging in a certain policy or activity (usingthese new washers) could result in a certain effect.” And if you go no further than that, only(D)'s conclusion fits that model. (A) describes a necessary consequence of a policy; (B)predicts a policy that a certain type of person will follow; (C) makes a deduction —describes what must be true — if a certain phenomenon occurs; and (E), close but no cigar,says that a certain policy can but won't necessarily have a certain effect. The original lacksthat sense of “not necessarily.” (D) must be correct.Of course, you could analyze a little deeper and recognize that the stimulus is reallywarning us of a possible paradox. It alleges that a particular policy may have the oppositeintended effect: New washers meant to be more energy-efficient may in fact use moreenergy. That makes it even clearer why (E) is wrong and (D) is right — there's no sense ofparadox in a Soloflex with heavier weights that may make you stronger, but it would beparadoxical to choose a furniture wood for its inexpensiveness and end up having tospend more money. Still and all, you don't even have to go into it that deeply. Try not todo any more work than you have to.

16. (E) It’s a two-question dialogue stimulus for Questions 16 and 17. Start with theshorter statement: H is astonished that there could have been bias, so before we even readit we know that G is making a bias accusation. And so she is. Allegedly this art show wasbiased in favor of photographers, based on the fact that more photos were exhibited thaneither sculptures or paintings even though equal numbers of photographers, painters, andsculptors submitted acceptable works. (For the purpose of this discussion, let’s use“acceptable” as a convenient euphemism for the passage’s “meeting the traditionalcriteria.”) H expostulates that bias was impossible, since every acceptable work submittedwas shown. This appears to be a paradox.

Page 57: Prep Test 9

KAPLAN LSAT PREP _______________________________________________________ LSAT Test IX Explained: Section IV

© K A P L A N 55

When a paradox or mystery is presented— even if, as here, the testmaker doesn’t announceit as such — it’s fun and efficient to try to figure out the solution yourself, before lookingat the choices. One answer would be that some people submitted both photos andpaintings or statues, but G has already thrown in a Logic Games-style “loophole closer” toeliminate that possibility. But you might also realize that nothing has been said about howmany acceptable works were submitted per medium or per person. If G is correct that thesame number of people in each medium submitted acceptable work, and if H is correctthat all acceptable work was exhibited, it has to be true that more photos were found to beacceptable than paintings or statues. And that’s what correct choice (E) says.Choices (A) and (B) all need not be true, because both try to draw conclusions about theworks submitted to the art show in general. But G and H are only concerned with thosesubmitted works that were acceptable. There could have been hundreds of submittedworks that didn’t meet the traditional criteria without having any effect on the argument athand. In the same vein, for all we know every submitted photo was deemed acceptable, so(D) need not be true; and the issue of quality (C) couldn’t be further from the minds of Gand H so it needn’t be considered by us.

17. (B) Generally, to strengthen an argument, we want to look for an additional piece ofevidence that shores up a key assumption. Choice (B) seems like a very clear show of biassince, if it’s cheaper to enter a photo in the contest than to enter a painting or a sculpture, aphotographer could afford to submit many more entries into the show than a painter orsculptor could. Choice (A) is way off, since it doesn’t make any distinction between thedifferent types of artists exhibiting, which is the crux of G’s allegation of bias. Even if thecommittee mentioned in (C) had been stacked with photographers, (C) would be adubious choice — remember, answer choices impugning people’s motives tend to bewrong answers — but the membership (C) actually describes is so inconclusive (in termsof which media predominates) that the choice shouldn’t have tempted you much. (D) goeseven further astray, having to do with the possible bias of the art show’s critics rather thanits governing board. And (E)’s art shows from previous years are beyond the scope of theargument between G and H, which is only about what happened this year.

18. (C) Anita strikes at the root of Marcus’ argument. He has denied that there’s anyproblem in applying traditional journalistic ethics to journalists’ typical ethical dilemmas— just print whatever’s newsworthy. Anita, somewhat condescendingly, points out thatMarcus’s formula basically begs the question, that it simply sidesteps the typical quandaryin which journalists find themselves: figuring out what is newsworthy and what isn’t. Herpoint is that Marcus’ “traditional journalistic” ethical principle will leave journalists highand dry in most dilemmas. That’s exactly what (C) says — when it comes down to helpinga journalist make one of his or her key decisions (“in a typical kind of situation in which adecision has to be made” as (C) says so elegantly), Marcus’ principle is useless.Anita doesn’t say that traditional journalistic ethics is unclear , as (A) would have it; shesays that the version given by Marcus is inadequate. (B) is probably the most temptingwrong choice. However, (B) implies that Anita has used an example to demonstrate thefailure of Marcus’ principle, but actually she merely defines what the journalist’s dilemmatypically is, and shows how Marcus’ principle is inadequate in such cases. Note, too, that(B) accuses Anita of denying that the traditional ethics are correct — but in point of factAnita denies that Marcus’ version of traditional journalistic ethics is adequate. As both (A)and (B) illustrate, taking careful note of the vocabulary used in dialogue questions makesit easier to choose between close alternatives. As for (D), Anita only attacks Marcus’

Page 58: Prep Test 9

KAPLAN LSAT PREP _______________________________________________________ LSAT Test IX Explained: Section IV

56 © K A P L A N

principle, she never claims that no principle can be useful. Finally, (E) is correct to adegree — Anita does imply that Marcus has simply described a journalist’s job when hesays that journalists should publish important information. However, this isn’t her point,it’s something she says on the way to her point, which is that his ethical principle isn’t ofmuch use.

19. (A) Question 19 asks us to tighten Anita’s argument. We want a necessary assumptionthat, together with Anita’s statements, would prove that Marcus’ claim about traditionaljournalistic ethics is wrong. In cases like this the first step is always to pinpoint theclaim being disputed. Marcus claimed that the traditional ethics is clear, adequate, andessentially correct, for most ethical dilemmas. Anita’s reply is that where a decision needs tobe made about whether a story is newsworthy, traditional journalistic ethics are inadequate.Well, in order for this reply to disprove Marcus’ contention we need to connect the termsused by the two protagonists. One would have to assume something like choice (A), thatthe question about whether something is or isn’t newsworthy can raise ethical dilemmas forjournalists. Otherwise, Anita’s reply would be irrelevant. You may have overlooked thissubtle assumption when reading Anita’s argument and doing Question 18, but you shouldhave recognized it when you started reading the answer choices.(B) is a tough one. Remember that Anita attacked traditional journalistic ethics as uselessto a journalist in deciding whether information is newsworthy. (B) says that even if an itemis newsworthy it may be unethical to publish it — which works against Anita’s statements,rather than with them, by suggesting that the question of whether or not information isnewsworthy is actually extraneous to the question of whether or not it ought to be printed.(B) says that the ethical dilemma does not have to do with newsworthiness, but lieselsewhere. (C) is beside the point; as we saw, Marcus’ claim had to do with ethicaldilemmas only. And in order to disprove Marcus’ claim that traditional journalistic ethicsis adequate for most ethical dilemmas, one need not assume either (D), that an adequatesystem of journalistic ethics would conclusively resolve any dilemma a journalist waslikely to face, or (E), that an adequate system of journalistic ethics would provide guidancein every situation. Remember, we’re looking for what must be assumed, not just astatement that sounds consistent with Anita’s point of view.

20. (C) The argument begins by going after burglar alarm systems, and those who usethem, hard: Almost all burglar alarms are false alarms, they take up dwindling policeresources, and an unfair share of police attention goes to alarm system users. Then, justbefore the conclusion, the author slips in this statement: Burglar alarm systems, unlike caralarm systems, do work. What’s the point of saying this?, asks Question 20. The dangerhere is getting hung up on car alarms, and thinking they must figure in the argumentsomehow, but that was only a parenthetical remark. What the statement is saying here issimply that burglar alarm systems work. So, after a long diatribe about the bad points of thesystems, the argument brings up a crucial point in their favor. Then comes the conclusionthat the only acceptable solution is to fine the offending systems owners. The statement thatburglar alarm systems work, then, is meant to lead to the conclusion that the only thing todo is to fine the owners. But those alarms are such a pain in the butt, what if they didn’twork? Well then, maybe the obvious conclusion would be to ban them altogether. So (C)accurately describes the role the statement plays: It gives a reason for not doing anythingdrastic, like banning alarms or refusing to answer them.(A) doesn’t work at all — burglar alarms are being compared favorably to car alarms. As for(B), the statement doesn’t suggest anything about the number of burglar alarms; anyway,

Page 59: Prep Test 9

KAPLAN LSAT PREP _______________________________________________________ LSAT Test IX Explained: Section IV

© K A P L A N 57

that’s a reference to sentence 1. Just as in Reading Comprehension, Logical Reasoninganswers can be wrong because they come from “the wrong part of the passage.” (D) fitsright in with the rest of the stimulus, explaining why burglar alarm systems are a drain onpolice resources, but the statement in question is going against the bulk of the paragraph.If you find that hard to see, look at the structural clues again — the statement is precededby a nice big “however.” In questions that ask what role a statement plays in anargument, take especially careful note of structuralclues — words like “however” and“although.” Finally, contrary to (E), the statement has nothing to do with businesses vs.homeowners.

21. (E) Another principle question — I hope you’re getting used to them. We want theprinciple that would justify the author in drawing the conclusion (that burglar alarmsystem owners should be fined for false alarms) from the premises (the various rottenaspects of such burglar alarm systems). It’s hard to prephrase a very precise answer here;the best you can do is probably something like, “people should pay for the trouble theycause,” and you know the answer is not going to be that vague. As usual in such cases,after making sure you understand the argument in question, the best thing is to gothrough the answer choices looking for a complete parallel, but always keeping the termsof the argument in mind. Again, those are: fining alarm owners; and the downside ofalarms.(A) misses the boat because it states that the practice in question should not be permitted,whereas the argument concludes that burglar alarm systems should be. (B) is tricky, butalarm system owners are not being sentenced (by the author) to pay extra in order toreceive special services, such as extra police surveillance in their neighborhoods; they’resupposed to pay damages for wasting police time. (C) is way off; in the first place, thepolice aren’t improving service to anyone, and in the second place, it’s never stated thatsome people’s possession of burglar alarms deters crime for their whole neighborhoods.Contrary to (D), the argument doesn’t suggest that people should reimburse the public forpolice services, only for wasting the police’s time with false alarms. (D) would mean thatpeople should also pay a fine for genuine alarms, which isn’t what the passage says at all.Finally, (E) covers all the bases: The burglar alarm system owners are receiving publicservices (police time), they are wasting public resources, they are taking up more than theirfair share of those services, and yes, they should compensate for the wasted resources.

22. (E) Another long argument for Question 22, and this is one example of an instancewhere it can really pay to read the question stem first. If you knew that you were lookingfor a weakness, it was probably much easier to plow through the text. People used to thinkthat butterfat was nutritious and healthful, and at that time a law was passed requiring thatbutter that had been diluted with water be labeled “imitation butter.” Now, the authorsays, we know that butterfat is unhealthy, so people should be encouraged to eat foodswith lower butterfat content. The term “imitation” deters people from buying dilutedbutter, so the law should allow this product to be labeled “lite butter,” a more appealingname.We can weaken this by showing that the author’s solution won’t help the problem, aschoice (E) does. (E) explains that the name “imitation butter” provides a service, becausepeople shy away from it and instead buy products with even lower butterfat contents.This is not so good for the imitation butter industry, but it undermines the author’sargument, and so it’s correct. Speaking of the industry, choice (A) tries to explain that thedecision to use the word “lite” was motivated by financial greed, but the reason for the

Page 60: Prep Test 9

KAPLAN LSAT PREP _______________________________________________________ LSAT Test IX Explained: Section IV

58 © K A P L A N

decision doesn’t have enough of an effect on the argument to be considered a weakener.As so often, the motivation behind a plan is not especially relevant to the wisdom of thatplan. Choice (B) would have the imitation butter makers watering their butter even more,which might help people eat even less butterfat, which is a good thing, so (B) strengthensthe author’s argument, if anything. (C) tells us that some people eat imitation butterdespite its name — well, that’s good, but it’s the people who are deterred from it andinstead eat real butter that the manufacturers are concerned with. And (D) focuses on thedetail of cholesterol in butter and its effects on health, which can’t weaken the argument —just because it’s not the only factor doesn’t mean it’s not an important factor, and it doesn’tmean that the author’s plan won’t work.

23. (E) We go from butterfat to farm animals with Question 23 — and we need a goodinference. According to our author, current farm management practices cause animalspain and distress because they impose behavior that is not in line with the animals’ naturalbehavioral tendencies. Farming practices that go along with the animals’ naturaltendencies, we’re told, can be more efficient, and can cause the animals less pain anddistress. It follows, then, as (E) states, that some changes in current practices that result inless pain and distress for the animals can also result in greater efficiency for farmers.Choice (A) is about altering the behavioral tendencies of the animals, a topic that is outsidethe scope, since the author is talking about altering the management practices and notaltering animal behavior. Choice (B) goes too far, since the author doesn’t imply that oneneeds to understand animal evolution in order to implement better management practices.(C) contradicts the stimulus, since we’re told that practices that are less painful anddistressful can be more efficient — choice (C) claims the opposite. Finally, choice (D) goestoo far, since we needn’t believe that the practices that cause the least amount of pain arethe most efficient — this is too strong to be inferable. For inference questions, always keepan eye out for wrong choices that go beyond the scope or contradict the stimulus.

24. (D) The question stem tells us we’re to find the choice that uses the same bad reasoningas the stimulus’; in other words, parallel reasoning. The author concludes that computersin the classroom were just a fad, that the prediction that they would play an important rolein education was incorrect. Her evidence for this is last year’s decline in sales ofcomputers for classroom use. For informal arguments like this one, try to “step back“from the argument and characterize it in a general way. Here, you might say somethinglike, “The author uses info from a recent year to conclude that a general trend has ended.”This is enough to get you to (D), which uses information from last year about microwaveoven sales to conclude that the microwave oven trend is over. Same argument, differentwords. By the way, although I didn’t discuss the major flaw in this argument, I hope yousaw it. Computers — and microwaves — are durable goods: once you buy one, you don’tneed to replace it for a long time. So the fact that computer sales — and microwave sales— have recently declined could actually mean that these items have already becomewidespread, that most classrooms now have computers and most households havemicrowaves, which would suggest that these items were not just a “fad.” So the author’sclaim that the alleged trend is over is premature at best.Rather than declaring a trend over, (A) talks about the success of government legislation, soit’s not parallel. Nor does (A) use info from a recent year — air pollution levels are thecumulative result of many years. (B)’s about as unlike the stimulus as you can get: insteadof declaring a trend over; it predicts that a breakthrough’s about to occur. Choice (C)’swhat we sometimes call a “same-field fooler”: the testmakers hope that you’ll fall for a

Page 61: Prep Test 9

KAPLAN LSAT PREP _______________________________________________________ LSAT Test IX Explained: Section IV

© K A P L A N 59

choice like this just because it uses the same subject matter — computer sales — as thestimulus argument. But the structure, which is what counts in parallel reasoning, istotally different. Instead of arguing that a fad is now past, (C) makes a prediction aboutthe future, claiming that home computer sales “will never” reach projected levels. Nor is(C)’s evidence anything like that in the stimulus. Finally, (E) does use evidence of a recentdecline, but unlike the stimulus argument and correct choice (D), (E) doesn’t use thisevidence to predict that a trend is over; it concludes that government efforts to createinvestment incentives cause a cycle of booms and busts.

25. (D) Don’t be scared off by the “scientific terminology” in questions like this one. Justtake the time to take it apart piece by piece and it’ll all come clear. A group ofexperienced researchers reported some controversial results from an experiment theyperformed Then a group of scientists tried to replicate the experiment, but failed to get thesame results. Thus, they concluded, the original results, the ones reported by theexperienced researchers, were due to faulty measurements. In order to conclude this, thescientists must be assuming that their replication attempt didn’t have the same problem asthe original experiment. Otherwise, how could they assume that it wasn’t their replicationattempt that failed? (D) makes this assumption concrete by explaining that the replicationresults weren’t as likely to be skewed by faulty measurements. If this were so, then thissecond attempt would be more reliable, and it would be more reasonable to conclude thatthe original experiment was the problematic one.Choice (A) puts the blame on the replication, but this doesn’t stay in line with the scientists’assertion, which claims that the fault lay in the original experiment. (B) stresses thecontroversiality of the results, but the scientists needn’t assume that this was the cause ofthe difficulty. (C)’s topic is the theory behind the original experiments, which is irrelevantto the comparison between the two sets of experiments. So is (E)’s reference to theresearchers’ having observed the results only once. (E) would have you suspect thatexperimental results achieved only once must be faulty, but whether the results wereobserved once in a blue moon or once per hour, that still has no necessary effect on theissue in question.

Page 62: Prep Test 9

KAPLAN LSAT PREP _______________________________________________________ LSAT Test IX Explained: Section IV

60 © K A P L A N

Here’s some suggested scratchwork that goes along with the explanations forGame 2:

___ ___ ___ ___ ___ ___ ___ ___

___ ___ ___ ___ ___ ___ ___ ___

___ ___ ___ ___ ___ ___ ___ ___

___ ___ ___ ___ ___ ___ ___ ___

___ ___ ___ ___ ___ ___ ___ ___

J K

J� N NK�

J➂ P PK➃

� �

➂ ➃

J N NKL M M/QL

J P PKL M L/M/Q Pick two

(Rule 1)

(Add Rule 2)

(AddRules 3 & 4)

...and Game 3:

1 2 3R__ R__ R__S__ S__ S__T__ T__ T__

k l mSAME

S k

Page 63: Prep Test 9

I.N. LL3097 Rev.A Printed in the USA